You are on page 1of 124

www.MRCPass.

com

Respiratory


elzohryxp@yahoo.com https://www.facebook.com/elzohryxp

May2012

www.MRCPass.com

Respiratory

RespiratoryQ001

Amanpresentswithsymptomssuggestiveofapneumonia.TheCXRconfirmsthis. Whichofthefollowingfeaturessuggestspoorprognosis?
A.Respiratoryrateof20 B.Temperature38C C.Age60 D.Ureaof10 E.MTSscoreof9outof10

Answer:d)ureaof10. TheCURB65scoreforpoorprognosisinpneumoniaare: *confusion(definedasanAMTof8orless) *ureagreaterthan7mmol/l *respiratoryrateof30breathsperminuteorgreater *bloodpressurelessthan90systolicordiastolicbloodpressure60orless *age65orolder


Dr.KhalidYusufElzohrySohagTeachingHospital2012 2

www.MRCPass.com

Respiratory

RespiratoryQ002
A60yearoldladypresentswithshortnessofbreath.AchestXrayconfirmsthatshehas arightsidedpleuraleffusion.AnAspiratewastakenandthesamplewassentforseveral tests. Whichoneofthefollowingisanindicationforachestdrain?
A.Highprotein B.Lowglucose C.LowLDH D.PH<7.2 E.Bloodstains

Answer:D)pH<7.2 Althoughahighproteinalsopointstowardsanexudates,alowpHisthebestmarkerof infectioninaneffusionrequiringchestdraininsertion.


Dr.KhalidYusufElzohrySohagTeachingHospital2012 3

www.MRCPass.com

Respiratory

RespiratoryQ3
A 75 year old heavy smoker presents to the hospital with breathlessness, wheezing and acoughwithyellowsputum. He has the following investigations: Hb 18 g/dl, WCC 12 x 10^9/l, ABGs show a pH of 7.38,pO2of8.5kPa,pCO2of7kPa. Whatisthediagnosis?
A.Bronchiectasis B.Chronicobstructivepulmonarydisease C.Mesothelioma D.Tuberculosis E.Cryptogenicfibrosingalveolitis

Answer:b)chronicobstructivepulmonarydisease. This patient has type II respiratory failure, without an acidosis, suggesting chronic CO2 retention.Healsoisaheavysmokerandhaspolycythaemia,makingCOPDmostlikely.
Dr.KhalidYusufElzohrySohagTeachingHospital2012 4

www w.MRCPass.c com

Res spiratory

Respiratory yQ4
A45 5yearoldman m presentswithach hroniccoughproductiv veofcopiou usamountsofthick yello ow sputum m and occasional haemoptysis. He e also has had multiple e episodes of o chest infe ection. On examinatio e c an nd on auscu ultation ove er the lung g bases, n he has clubbing coarsecrepitat tionsarehe eard. ichinvestigationwillre evealthedi iagnosis? Whi
A.Chrom mosometest ting B.Bronc choscopy C.Sweat ttest D.Maxil llarysinusxrays E.Highresolution r CT T

Answer:e)Highresolution nCT. The e diagnosis is bronchie ectasis. HRCT is likely y to show bronchial dilatation d and wall thic ckening.

HRC CTshowingbronchiectasis

Dr.Khalid K Yusuf fElzohrySo ohagTeachin ngHospital 2012 5

www w.MRCPass.c com

Res spiratory

Respiratory yQ5
A40 0yearoldman m hasem mphysema.H Heisfoundtohavethe eSSphenot type. atishisalpha1antitry ypsinlevelli ikelytobe? ? Wha
A.10% B.15% C.25% D.50% E.75%

Answer:D)50% %. The e normalgenotype isMM M andleve els of enzym me is(100% %).The relev vant enzym melevels areMS(75%),MZ(55%),SS S (50%)ZZ(15%.)

Dr.Khalid K Yusuf fElzohrySo ohagTeachin ngHospital 2012 6

www.MRCPass.com

Respiratory

RespiratoryQ6
A 70 year old man with known chronic obstructive pulmonary disease (COPD) is admitted with symptoms of worsening breathlessness and confusion. Following a dose of 200mg iv hydrocortisone and two doses of salbutamol and atrovent nebulisers, arterialbloodgasesweretaken. Hisarterialbloodgasesareasfollows:pH7.18,PO26.6kPa,PCO212.0kPa. Whatshouldbedonenext?
A.Repeatdoseofcorticosteroids B.Mechanicalventilation C.2litresoxygen D.Monteleukast E.Magnesiuminfusion2g

Answer:b)Mechanicalventilation. ThispatienthastypeIIrespiratoryfailurewithacidosis,andhenceshouldbeconsidered forventilation,eitherstraightawayorwithatrialofNIPPVbeforehand.


Dr.KhalidYusufElzohrySohagTeachingHospital2012 7

www.MRCPass.com

Respiratory

RespiratoryQ7
A 40 year old woman has been breathless and is undergoing investigations for the cause. She has a previous history of being a smoker and has a dry cough as well. Her ABGs show a pO2 of 9 kPa and pCO2 of 5 kPa. Lung function tests show reduced vital capacity,reducedTLCObutincreasedKCO. Whatisthelikelydiagnosis?
A.COPD B.Bronchialobstructionfromatumour C.Kyphoscoliosis D.Obstructivesleepapnoea E.Fibrosingalveolitis

Answer:c)kyphoscoliosis. In this scenario, the lung function indicates that the surface area for exchange are inadequate (low TLCO) hence suggesting that lung expansion is reduced. How ever, cardiacoutputisincreasedtocompensate,andhencedKCOisincreased.Kyphoscoliosis wouldrestrictlungexpansion,oraneuromusculardisorder(egspinabifida)couldcause this.
Dr.KhalidYusufElzohrySohagTeachingHospital2012 8

www.MRCPass.com

Respiratory

RespiratoryQ8
A50yearoldwomanisadmittedwithinfectiveexacerbationofasthma.Sherespondsto medicaltreatmentbutAspergillusfumigatusiseventuallyculturedfromhersputum. FurtherinvestigationsshowserumtotalIgElevelwaselevatedat350ng/ml(normal40 180 ng/ml),RAST to Aspergillus fumigatus was class III, Aspergillus fumigatus precipitins arenegative. Whatisthemostappropriatemanagement?
A.Nochangeinmedication B.Highdoseoralcorticosteroids C.CTscanofthechest D.ChestXray E.Itraconazole

Answer:a)Nochangeinmedication. With negative Aspergillus fumigatus precipitins and serum total IgE less than 1000 ng/ml,thispatientisunlikelytohaveAllergicBronchopulmonaryAspergillosis(ABPA). FeatureswhicharefoundcommonlyinasthmaticswithoutABPAinclude: Positive immediate skin reactivity to Aspergillus fumigatus, which is present in 2030% ofasthmaticsPositiveserumprecipitinstoAspergillus,whichoccurin10%ofasthmatics withoutABPARecurrentmucoidimpactionandatelectasisPeripheralbloodeosinophilia andelevationofserumtotalIgE.
Dr.KhalidYusufElzohrySohagTeachingHospital2012 9

www.MRCPass.com

Respiratory

RespiratoryQ9
A 60 year old man presents with small amounts of haemoptysis and breathlessness. He doesnothavepleuriticchestpainsandhasnormaloxygensaturations. In considering the potential diagnosis, haemoptysis can occur with which of the followingdiagnosis?
A.Pulmonaryfibrosis B.Melanoma C.Goitre D.Thymoma E.Aspergilloma

Answer:e)Aspergilloma. Haemoptysiscanbecausedbypulmonaryembolus,tuberculousinfection,aspergilloma, bronchialcarcinoma,Goodpasture'ssyndromeandWegener'sgranulomatosis.


Dr.KhalidYusufElzohrySohagTeachingHospital2012 10

www.MRCPass.com

Respiratory

RespiratoryQ10
A 30 year old mangives a sixmonth history of worsening breathlessness and coughs up half a cupful of sputum daily. He was occasionally wheezy with viral illnesses as a young child,worksinafactoryandhassmoked25cigarettesperdayforthelastfouryears. Themostlikelydiagnosisis:
A.Lungcarcinoma B.Bronchiectasis C.AsthmaAsbestosis D.Chronicobstructivepulmonarydisease E.Rheumatoidlung

Answer:b)Bronchiectasis. Bronchiectasis is most likely due to the extensive amounts of sputum production. In a youngperson,cysticfibrosisandhypogammaglobulinaemiashouldbeconsidered.
Dr.KhalidYusufElzohrySohagTeachingHospital2012 11

www.MRCPass.com

Respiratory

RespiratoryQ11
A 60 year old man has smoked for 10 years. He has a longstanding dry cough and expiratorywheeze. Examinationrevealsscatteredrhonchiandreducedlungexpansion. Themostlikelydiagnosisis:
A.Asthma B.Lungcarcinoma C.Chronicbronchitis D.Bronchiectasis E.Extrinsicallergicalveolitis

Answer:c)chronicbronchitis. Theheavysmokinghistoryandwheezesuggestschronicbronchitis.
Dr.KhalidYusufElzohrySohagTeachingHospital2012 12

www.MRCPass.com

Respiratory

RespiratoryQ12
A 14 year old boy develops red eye and rhinitis frequently during the start of the summer. Whatisthelikelytriggeringagent?
A.Housedustmite B.Grasspollen C.Willowpollen D.Isocyanates E.Coaldust

Answer:B)grasspollen. Grass pollen is released late May till August, and willow pollen is released from March tillJune.Astheallergicrhinitisistriggeredduringthestartofsummer,thisismostlikely tobeduetograsspollen.
Dr.KhalidYusufElzohrySohagTeachingHospital2012 13

www.MRCPass.com

Respiratory

RespiratoryQ13
A65year old manpresents withbreathlessnessthathas gotgraduallyworseover three months. He has longstanding atrial fibrillation, for which he takes warfarin and amiodarone. On examination his pulse is 100/min in AF, oxygen saturation was 90% on air. His JVP was not raised and he has fine bibasal crackles. Arterial blood gas saturation showed mildhypoxiaandapulmonaryfunctiontestrevealedamoderaterestrictivepicture. Themostlikelydiagnosisis:
A.Bronchiectasis B.Pulmonaryembolism C.Pulmonaryhaemorrhage D.Amiodaroneinducedinterstitiallungdisease E.Congestivecardiacfailure

Answer:d)amiodaroneinducedinterstitiallungdisease. The case scenario would fit amiodarone induced interstitial lung disease or pulmonary fibrosis. This may take several months or years to develop. Lung function tests may show a restrictive picture with reduced transfer factor. A high resolution CT in this case islikelytoshowdiffuse"groundglass"opacitiesorinterlobularseptalthickening.
Dr.KhalidYusufElzohrySohagTeachingHospital2012 14

www.MRCPass.com

Respiratory

RespiratoryQ14
A65yearoldmanhasbeenconfirmedtohavelungcancer. Whichformofcancerhastheworstprognosis?
A.Smallcell B.Squamouscell C.Adenocarcinoma D.Largecell E.Prostatemetastasis

Answer:a)smallcell. Small cell lung cancer has the worst prognosis out of all lung cancers and is rarely suitable for surgical resection. Even when diagnosed early the 2year survival is in the orderof2025%.Themainformoftherapyischemotherapy. Small cell cancer is associated with syndrome of inappropriate antidiuretic hormone (ADH) and squamous cell cancer is associated with paraneoplastic hypercalcaemia through increased parathyroidrelated hormone. 75% of small cell cancers arise in the proximalairways.
Dr.KhalidYusufElzohrySohagTeachingHospital2012 15

www.MRCPass.com

Respiratory

RespiratoryQ15
A 22 year old female presents with a chest infection. She is unable to complete a sentence and her peak flow rate was 40% of her normal level. She is treated with high flowoxygen,nebulisedbronchodilatorsandoralsteroidsbutthisisassociatedwithlittle changeinhercondition. Which of the following treatments, given intravenously, would be the most appropriate forthispatient?
A.Aminophylline B.Augmentin C.Hydrocortisone D.Magnesium E.Salbutamol

Answer:D)Magnesium. IVmagnesiumisrecommendedbytheBritishThoracicSocietyinsevereexacerbationof asthma. A dose of 2g (8 mmol) is given as a bolus iv dose in the acute presentation period.
Dr.KhalidYusufElzohrySohagTeachingHospital2012 16

www.MRCPass.com

Respiratory

RespiratoryQ16
A60yearmanhasbeenasmokerof20adayfor30years.Hehasacoughanddifficulty in breathing for 3 days which has worsened and he presents to hospital. He has home nebulisersbutnothomeoxygen. On examination, his oxygen saturations are 95%, blood pressure 110 / 60 mmHg and respiratory rate32. Hehasdiffuse wheezesand isusing hisrespiratorymuscles. Arterial bloodgasesdoneon6litresofoxygenshow: pH7.25 P0216kPa PCO28.2kPa Whatshouldbethenextmanagementstep?
A.Noninvasiveventilation B.Intravenousantibiotics C.Intubationandventilation D.Reduceinspiredoxygenconcentration E.Increaseoxygenconcentration

Answer:d)reduceinspiredoxygenconcentration. Thismanwith ChronicObstructivePulmonaryDisease (COPD)hasbloodgasesshowing type II respiratory failure with acidosis. He has been placed on 6 litres of oxygen, which is too much for a patient with severe COPD as the history of home nebuliser use suggests. Hisrespiratorydriveissuppressedbytoomuchoxygeninspired,andhencereductionto a lower concentration (e.g. 1 2 litres) to maintain a pO2 above 8.5 kPa is recommended.
Dr.KhalidYusufElzohrySohagTeachingHospital2012 17

www.MRCPass.com

Respiratory

RespiratoryQ17
A 70 year woman has a history of dry cough for 2 months. She has lost 5 kg of weight over the 2 months. Her chest X ray shows a left apical shadowing. Blood tests reveal a raisedwhitecellcountof16.Shehasnotmanagedtocoughupanysputum. Whichtestshouldbeperformed?
A.CTscanofthechest B.SerumANCA C.Ultrasoundofthechest D.Kveimtest E.Bronchoscopy

Answer:e)bronchoscopy. This patient is likely to have TB due to the apical shadowing. In a patient who is unable to expectorate sputum, bronchoscopy with lavage (send for AFB) should be performed toconfirmthediagnosis.
Dr.KhalidYusufElzohrySohagTeachingHospital2012 18

www.MRCPass.com

Respiratory

RespiratoryQ18
A 65 year old man with emphysema presented with increasing dyspnoea and leftsided pleuritic chest pain. A chest radiograph reveals a leftsided pneumothorax with a lung edgemeasured5cmawayfromthechestwall. Themostappropriatemanagementofhisconditionis:
A.Reviewwithdailychestradiographs B.Intercostaltubedrainage C.Refertoarespiratoryoutpatientclinic D.CTchest E.Simpleaspiration

Answer:b)Intercostaltubedrainage. The lung edge measurement suggests a greater than 50% pneumothorax. A smaller pneumothorax may be amenable to aspiration. In the context of chronic underlying respiratory disease such as chronic obstructive pulmonary disorder and a large pneumothorax,patientsarebestmanagedbypleuraldrainage.
Dr.KhalidYusufElzohrySohagTeachingHospital2012 19

www.MRCPass.com

Respiratory

RespiratoryQ19
A 70 year old man attends the hospital with a history of proximal muscle weakness. He also gives a history of cough of 12 weeks duration and complains of pain of the small joints of the hands. He has small haemorrhages in the nail folds, but is not clubbed. On examination of the chest he has bibasal crackles, and a chest radiograph reveals diffuse reticularinfiltrates.Lungfunctiontestsconfirmarestrictivepattern. Whatisthelikelycauseofhisinterstitiallungdisease?
A.SLE B.Dermatomyositis C.Ankylosingspondylitis D.Cryptogenicfibrosingalveolitis E.Amiodaroneinducedfibrosis

Answer:b)Dermatomyositis. Inpolymyositisanddermatomyositispatientsoftendevelopproximalmuscleweakness and of pain in the small joints of the fingers. They may have ragged cuticles and haemorrhages at the finger nail folds. Interstitial lung disease can occur. Underlying malignancy(lungs,ovaries,breastsandstomach)ispresentin5%ofcases.
Dr.KhalidYusufElzohrySohagTeachingHospital2012 20

www w.MRCPass.c com

Res spiratory

Respiratory yQ20
A 40 4 year old d man is re eferred for investigati ion of the cause of chronic c cou ugh. He desc cribes long standing s sputum pro oduction wh hich is strea aked with blood. There is no feve erornightsweats, s and dhedoesno otsmoke.How H everth hereisafam milyhistoryofsuch sym mptoms.Che estXraysho owsmildbi ilaterallowe erzoneshadowing. ichisthemostappropr riateinvesti igation? Whi
A.HighresolutionCT T B.Exerc cisetolerance etest C.Echoc cardiography y D.Bronc choscopyandbiopsy E.Serum mprecipitins

Answer:a)high hresolution nCT. The e likely diagnosis is bro onchiectasis s, and a fam mily history is suggestiv ve of cystic fibrosis or K Kartagener's syndrome e. HRCT wil ll help to co onfirm the diagnosis, if changes such as dilatedcentraltubularbro onchiandm mosaicoligaemiawereseen.

Bronchiectasis AirwaydilatationontheHRCT

Dr.Khalid K Yusuf fElzohrySo ohagTeachin ngHospital 2012 21

www.MRCPass.com

Respiratory

RespiratoryQ21
A 40 year old man has a 3 month history of cough and dyspnoea. He smokes 20 a day. Thereisnohistoryofasbestosexposure. HisWBCcountis24x10^9/lwith70%neutrophilsand3.0x10^9/l(5%)eosinophils.IgE leveliselevated. HehasdecreasedbreathsoundscorrespondingtoparenchymalinfiltratesontheCXR. Whichisthebesttesttoconfirmthediagnosis?
A.SputumforAcidFastBacilli B.HIVtest C.Autoimmunescreen D.AspergillusRAST E.Stoolforova,cysts,parasites

Answer:d)AspergillusRAST. The condition described is Allergic Broncho Pulmonary Aspergillosis, which is commoner among asthmatics and cystic fibrosis patients. Eosinophilia and high IgE levels are suggestive of this condition. RAST test for antibodies tow ards Aspergillus confirmsthediagnosis.
Dr.KhalidYusufElzohrySohagTeachingHospital2012 22

www.MRCPass.com

Respiratory

RespiratoryQ22
A45yearoldwomanisadmittedwitha2dayhistoryoffever,rigorsandbreathlessness. She looks extremely unwell and is confused, cyanosed, has a respiratory rate of 24/min andasystolicbloodpressureof85mmHg. There is bronchial breathing at her right base, where a chest radiograph reveals consolidation. Whichwouldbethemostappropriateantibioticregimen?
A.Intravenousceftazidimeandintravenousgentamicin B.Oralerythromycin C.Intravenouscefotaximeandoralerythromycin D.Intravenousamoxicillinandoralclarithromycin E.Oralamoxicillinandoralclarithromycin

Answer:c)Intravenouscefotaximeandoralerythromycin. Severe pneumonia as defined by the British Thoracic Society guidelines, is diagnosed when there are two of the following features: confusion, urea >7mmol/l, respiratory rate>30/min,andhypotension(SBP<90mmHg,DBP<60mmHg). Appropriate treatment is with intravenous antimicrobials: cefuroxime 1.5g three times dailyorcefotaxime1gthreetimesdailydailyPLUSerythromycin500mgfourtimesdaily orclarithromycin500mgtwicedaily.
Dr.KhalidYusufElzohrySohagTeachingHospital2012 23

www.MRCPass.com

Respiratory

RespiratoryQ23
A 72 year old man presents with a history of worsening breathlessness and cough. His arterialbloodgasesshowthefollowingrecordingswhentakenatroomair: pH7.25 pO26.4kPa pCO28.9kPa Bicarbonate31mmol/L Whatisthemostlikelydiagnosis?
A.Acuteexacerbationofchronicobstructivepulmonarydisease B.Obstructivesleepapnoea C.Pulmonaryoedema D.Pulmonaryembolus E.Pulmonaryfibrosis

Answer:a)Acuteexacerbationofchronicobstructivepulmonarydisease. The blood gas result would be most compatible with a patient with severe COPD and chronic type 2 respiratory failure with an acute exacerbation (Hypoxia, respiratory acidodiswithraisedCO2andmetaboliccompensation).
Dr.KhalidYusufElzohrySohagTeachingHospital2012 24

www.MRCPass.com

Respiratory

RespiratoryQ24
A 70 year old man presents with a chronic cough. He is a heavy smoker of over 40 cigarettes a day. CXR shows a peripheral rightsided lesion. A biopsy which was taken shows squamous cell carcinoma. No regional lymph nodes are involved. Lung function testsshowaFEV1oflessthan1.5litres. Therecommendedtreatmentis:
A.Chemotherapy B.Radiotherapy C.Lobectomy D.Pneumectomy E.Lungtransplant

Answer:b)Radiotherapy. Although surgical treatment is possible in non small cell lung carcinoma, a FEV1 of less than1.5litrescontraindicatessurgery.Hence,radiotherapyisrecommended.
Dr.KhalidYusufElzohrySohagTeachingHospital2012 25

www.MRCPass.com

Respiratory

RespiratoryQ25
A 55 year old gentleman has pickw ickian syndrome. He has poor exercise tolerance of 50yardsandoftenfeelslethargicatwork. Whichofthefollowingisthebestinvestigation?
A.Echocardiographytoassesscorpulmonale B.CTscanofthechest C.Bloodgas D.Sleepstudy E.Exercisetolerancetest

Answer:d)sleepstudy. The diagnosis of obstructive sleep apnoea can be made with a sleep study (polysomnography). In sleep apnoea, there is gross obesity and airways obstruction, occasionally leading to type II respiratory failure. During the sleep study, > 10 episodes of apneic episodes (pauses in breathing) satisfies the criteria for obstructive sleep apnoea.
Dr.KhalidYusufElzohrySohagTeachingHospital2012 26

www.MRCPass.com

Respiratory

RespiratoryQ26
A60yearoldmansmokerof35packyearspresentswitha6monthhistoryofshortness ofbreath.Hispastmedicalhistoryincludesdiabetesandcervicalspondylosis. SpirometryshowsFEV1of1litre65%predicted FVC1.03litres57%predicted FEV1/FVCratioof95. Howwouldyouinterpretthespirometryresults?
A.Normal B.Mixeddefect C.Obstructivedefect D.Restrictivedefect E.Suggestiveofhaemorrhage

Answer:d)Restrictivedefect. ReducedFEV1andFVCwithnormalFEV1ratioiscompatiblewithrestrictivedefect. Causesofrestrictivelungdefectare: neurogenicorpsychogeniccauses abnormalitiesofthethoracicwall stiffparenchyma(pulmonaryfibrosis) lossoflungtissue,e.g.pneumonectomy displacement


Dr.KhalidYusufElzohrySohagTeachingHospital2012 27

www.MRCPass.com

Respiratory

RespiratoryQ27
A 65 year old man has worsening breathlessness. He has a past medical history of arthritisandpalpitations.Onadmissionhisoxygensaturationswere90%onair. ABGsshowpO29kPa.pCO23.5kPa,pH7.36. HischestXrayshowspatchyshadowinginbothlungperipheries. Whichoneofthefollowingdrugsislikelytoberesponsible?
A.Codeine B.Tramadol C.Simvastatin D.Amiodarone E.Prednisolone

Answer:d)amiodarone. The hypoxia and X ray changes suggest pulmonary fibrosis. Out of the list of medications, amiodarone is the most likely candidate. A lung function test with transfer factor and also high resolution CT will help to confirm the diagnosis of amiodarone relatedpulmonaryfibrosis.
Dr.KhalidYusufElzohrySohagTeachingHospital2012 28

www.MRCPass.com

Respiratory

RespiratoryQ28
A 65 year old man has been diagnosed with chronic obstructive pulmonary disease (COPD).SpirometryconfirmssevereCOPDwithaFEV1oflessthan25%predicted.Inthe lastyearhehasbeenadmittedtohospitalon6occasionswithCOPDexacerbation. Whichoneofthefollowingcanhelptoreducehospitaladmissions?
A.Tiotropium B.Monteleukast C.Oraltheophyllines D.Salmeterol E.Hydrocortisone

Answer:a)Tiotropium. Severe COPD is diagnosed if the FEV1 is less or equal to 30% predicted. Studies have shown that patients treated with long acting anticholinergic (e.g. tiotropium) have few erexacerbationsperyear.
Dr.KhalidYusufElzohrySohagTeachingHospital2012 29

www.MRCPass.com

Respiratory

RespiratoryQ29
An 13 year old male has recent onset breathlessness. He has a history of wheezing particularlyduringthesummerwhenthepollencountishigh. Whichofthesetestswouldhelptoconfirmthediagnosis?
A.Trialofinhaledcorticosteroids B.Trialofanticholinergics C.Serialpeakflowmeasurements D.Lungfunctiontests E.ChestXray

Answer:c)serialpeakflowmeasurements. The history of wheezing during pollen exposure suggests asthma. The best diagnostic testforasthmawouldbedemonstrationofvariableairwaysobstructionwithserialpeak flow measurements. Asthmatic patients with exacerbation will demonstrate peak flows lowerthantheirpredictedpeakflow,andalsoamorningdip.Thereisalsoreversibilityif abronchodilator(e.g.salbutamol)isadministered.
Dr.KhalidYusufElzohrySohagTeachingHospital2012 30

www.MRCPass.com

Respiratory

RespiratoryQ30
A 27 year old man presents with sudden onset left sided pleuritic chest pain and breathlessness. Onexaminationheisdistressed,tachypnoeicandhastrachealdisplacementtotheright together with a hyperresonant percussion note on the left side. There were no breath soundsontheleft.Chestxrayshowsa<2cmleftsidedpneumothorax. Whatisthebestmanagementprocedure?
A.Highdoseoxygen B.Intercostaldrainagetube C.Chemicalpleurodesis D.Aspirationofpneumothorax E.Repeatchestxrayin6hours

Answer:d)aspirationofpneumothorax. Treatmentofapneumothoraxof<2cmmarginwouldbeinitialaspiration. This is less painful, leads to a shorter duration of admission, reduces the need for pleurectomy.
Dr.KhalidYusufElzohrySohagTeachingHospital2012 31

www.MRCPass.com

Respiratory

RespiratoryQ31
A 65 year year old lady presents with weight loss and haemoptysis and is admitted to hospital. She has been a heavy smoker for 25 years. A Chest X ray shows a mass in the rightupperlobe. Whichisthebesttesttoinvestigateatpresent?
A.Sputumcytology B.CTscanofthelung C.Transbronchialbiopsy D.Lymphnodebiopsy E.Pleuralbiopsy

Answer:a)sputumcytology. The mass seen on the CXR is likely to be a malignancy in view of the history of smoking, haemoptysis and weight loss. The initial test should be sputum cytology, as the diagnosis can be confirmed most easily. Staging CT scans can then be done to guide furthermanagementafterthis.
Dr.KhalidYusufElzohrySohagTeachingHospital2012 32

www w.MRCPass.c com

Res spiratory

Respiratory yQ32
A 70 0 year old man has a diagnosis o of non small cell lung tumour, t and has comp pleted a setofinvestiga ations. Whi ichoneofth hefollowing gisacontra aindicationtolungsur rgery?
A.FEV1of1.7L(50% %predicted) B.Horne erssyndrom me C.Historyofmyocar rdialinfarctio on D.Hypercalcaemia E.Neuro opathyaffect tinglowerlim mbs

Answer:B)Hor rnerssyndr rome. An FEV1 of < 1.1 1 L is a co ontraindicat tion for mo ost cardioth horacic surg gical proced dures. A malignant pleu ural effusio on, distant metastases s, contralateral mediastinal lymp ph node spre ead, vocal cord paralysis, phren nic nerve paralysis, p Horner's H sy yndrome, and SVC synd dromearecontraindic c ationstosu urgeryinlun ngcancer.

Left tsidedHorn ner's


Dr.Khalid K Yusuf fElzohrySo ohagTeachin ngHospital 2012 33

www.MRCPass.com

Respiratory

RespiratoryQ33
A20yearoldmalepresentswithbreathlessnessandwheezing. Whichofthefollowingismostlikelytosuggestasthma?
A.IncreasedserumIgE B.Wheezinginducedbysmoking C.Obstructivepictureinthelungfunctiontests D.Responsetoprednisolone E.DiurnalPEFRvariation>20%

Answer:e)diurnalPEFRvariationof>20%. Inasthma,diurnalPEFRvariabilityisduetovariousdegreesofbronchialhyperreactivity. This is the best indicator of likely asthma. A raised IgE indicates atopy but is not diagnosticofasthma.
Dr.KhalidYusufElzohrySohagTeachingHospital2012 34

www.MRCPass.com

Respiratory

RespiratoryQ34
A 70 year old patient with COPD presents with cough and breathlessness which has worsenedover2months. Onadmission,hehasthefollowingarterialbloodgasresults. pH7.32,pO27kPa,pCO28kPa,HCO334mmol/l,BaseExcess1. Whichoneofthefollowingbiochemicalstatesfitsbest?
A.Chronicrespiratoryacidosis B.Chronicrespiratoryalkalosis C.Chronicmetabolicacidosis D.Chronicmetabolicalkalosis E.Acutemetabolicacidosis

Answer:a)chronicrespiratoryacidosis. There is a mild respiratory acidosis (pH < 3.5 and pCO2 > 6) and base excess is also not elevated (in this case only1). There is metabolic compensation, as indicated by a high bicarbonate ( >30). The patient's history of COPD suggests that he has chronic hypoxia andchronicCO2retention,hencecausingthepictureofchronicrespiratoryacidosis.
Dr.KhalidYusufElzohrySohagTeachingHospital2012 35

www.MRCPass.com

Respiratory

RespiratoryQ35
Abreathless35yearoldwomanhasthefollowinglungfunctiontests: FEV11.2L(65%) FVC1.4L(60%) FEV1/FVCratio=82%predicted TLC=65%predicted RV=60%predicted TLCO=57%predicted KCO=105%predicted Whatisthemostlikelydiagnosis?
A.COPD B.Bronchiectasis C.Cysticfibrosis D.Scoliosis E.Pneumonia

Answer:d)Scoliosis. The lung function tests show a significant restrictive defect. Only kyphoscoliosis or a pneumonitis may fit this picture but given the normal/high KCO (i.e. after correcting for alveolar volumes), the most likely answer is kyphoscoliosis as the gas exchange after correctingforthealveolarvolumeishigh.
Dr.KhalidYusufElzohrySohagTeachingHospital2012 36

www.MRCPass.com

Respiratory

RespiratoryQ36
A70yearoldmanpresentedwithshortnessofbreath.Onexamination,hehadthesigns ofalargerightsidedpleuraleffusion. Investigationsrevealed:Pleuralfluidanalysisprotein65g/L Whatisthemostlikelycause?
A.Congestivecardiacfailure B.Nephroticsyndrome C.Nonspecificpericarditis D.Livercirrhosis E.Mesothelioma

Answer:e)Mesothelioma. It is a case of exudative pleural effusion (protein >30 g/l). Mesothelioma is the most likely cause in this case. Other causes are: malignancies, infection, autoimmune conditions.
Dr.KhalidYusufElzohrySohagTeachingHospital2012 37

www.MRCPass.com

Respiratory

RespiratoryQ37
A 75 year old man with COPD is on longterm oxygen therapy (LTOT). He complains of persistentlegswellingduringaroutinereview. HisABGonasupplementaloxygenflowrateof2l/min,shows: pHof7.35,pCO2of5.7kPa,pO2of7.8kPaandHCO3of28mmHg Whatshouldbedone?
A.CXR B.Overnightoxygensaturationmonitoring C.Echocardiogram D.RepeatABGonair E.CTscanofthechest

Answer:b)Overnightoxygensaturationmonitoring. The presence of persistent oedema or secondary polycythaemia suggests that the correctionofovernightSaO2maybeinadequate.Nocturnalhypoxaemiamaybeevident duringanovernightSaO2monitoring.
Dr.KhalidYusufElzohrySohagTeachingHospital2012 38

www w.MRCPass.c com

Res spiratory

Respiratory yQ38
A 5 50 year man presents with a six x month hi istory of co ough and breathlessn b ess. On examinationhe ehasclubbingandthe erearebilat teralfineins spiratorycr racklesinth hebases h lungs. ofhis A ch hest X ray reveals r bilat teral basal s shadowing in the lung fields. His blood gases reveal hypoxiawithapO2of9kP Pa. atisthebes stinvestiga ationtoconf firmadiagn nosis? Wha
A.Lungfunctiontests B.Trans sbronchialbiopsy C.Serum mACElevel D.HighresolutionCToftheches st E.Serum mprecipitins

Answer:d)high hresolution nCTofthechest. c The e diagnosis is likely to o be pulmo onary fibro osis. The signs clubbin ng and inspiratory crep pitations su uggest cryp ptogenic fib brosing alve eolitis. High h resolution CT would show retic cular opacities or hon neycomb ch hanges if th here is pulm monary fibrosis. HRCT T of the chestshowsba asilargroundglassopacitiesaswe ellaslinearandreticula aropacities s

HRC CT of the ch hest showin ng basilar ground glass s opacities as well as linear and reticular r opacitiesconsistentwith~ ~Pulmonary yFibrosis

Dr.Khalid K Yusuf fElzohrySo ohagTeachin ngHospital 2012 39

www w.MRCPass.c com

Res spiratory

Respiratory yQ39
A 40 0 year old man m presen nted with h history of re ecurrent Ha aemoptysis. He has pre eviously bee en treated for f tuberculosis. Chest t x ray show wed a roun nded soft tissue mass is seen with hinacavityintheright tupperlobe ewithanaircrescent. atisthedia agnosis? Wha
A.Chlam mydiapneum monia B.Capla an'ssyndrom me C.Extrin nsicallergica alveolitis D.Asper rgilloma E.Legion nellainfectio on

Answer:d)Asp pergilloma. Asp pergillomas are masse es of funga al mycelia that grow in preexis sting lung cavities. c Hae emoptysis is a commo on symptom m. They may m require treatment t with either anti fung gals or excision surger ry if large. They T are no ot associate ed with bro onchiectasis s (unlike allergicbronchopulmonar ryaspergillo osis).

Aspergilloma

Dr.Khalid K Yusuf fElzohrySo ohagTeachin ngHospital 2012 40

www w.MRCPass.c com

Res spiratory

Respiratory yQ40
A 55 5 year old d man who o had two episodes of o hemorrh hagic shock k due to in ntestinal hem morrhage an nd postope erative seco ondary hem morrhage, was w admitted to intensive care unit t. Dur ring the following weeks this was followed by b bronchop pneumonia with symptoms of seps sis persistin ng over sev veral weeks s. Chest x ray showed d progressi ive changes s in the inte erstitialtissu uesandhebecamemo oresignifica antlyhypoxic.Pulmona arycapillary ywedge pres ssurewas13 1 mmHg. atisthedia agnosis? Wha
A.Conge estivecardia acfailure B.Pneum mocystispne eumonia C.Pulmo onaryembol lism D.Adult trespiratorydistresssyndrome E.Comm munityacquiredpneumo onia

Answer:d)adu ultrespirato orydistresssyndrome. Respiratory di istress syndrome is associated with profo ound hypoxia and increased vasc cular permeability (ca ausing a V/ /Q mismatc ch). A norm mal PCWP differentia ates the conditionfrom mpulmonary yoedema.Itdoesnotrespondtosteroids.

ARD DS

Dr.Khalid K Yusuf fElzohrySo ohagTeachin ngHospital 2012 41

www.MRCPass.com

Respiratory

RespiratoryQ41
A40yearoldmanisHIVpositive.Hepresentswithbreathlessnesstothehospital.Chest XR shows bilateral interstitial lung markings. His oxygen levels desaturate upon mobilising. Whatshouldbecommenced?
A.Amoxycillin B.Amoxycillinandclarithromycin C.Ivcotrimoxazole D.Doxycycline E.QuadrupleTBtherapy

Answer:c)ivcotrimoxazole. The diagnosis is PCP. Silver staining rather than auramine should be done. Mortality is 10%.Ivcotrimoxazoleorpentamidinecanbeused.Desaturationistypicalwithexercise inPCP.OtherbacterialinfectionssuchasTBaremorecommonamongAfricans.
Dr.KhalidYusufElzohrySohagTeachingHospital2012 42

www w.MRCPass.c com

Res spiratory

Respiratory yQ42
A 25 year old male with h a 5 year long histor ry of bronc chial asthm ma, presente ed with wor rsening of asthma a and fever of 20 0 days dura ation. The patient p was well contro olled on bud desonide metered dos se inhalatio ons (400 g/day) g and rarely ever required to take salb butamolinh halations. For the past 20 2 days, th he patient was w experie encing incr reasing breathlessness s, fever, ductionofmucoid m expectorationforthepast t20days. coughandprod 80mmHg,a andtemper rature38.2 C. ExaminationaBPof130/8 aminationrevealed r bilateralwide espreadwhe eezes. Respiratoryexa estigations revealed Hemoglobi in 12.4 g/ /dl, total white w coun nt 13 x 10^9/l, Inve Neu utrophils 6 (27.5) x 10^9/l, Lymphocytes 3 (1.33.5) x 10^9/l, Eo osinophils 4 (0.04 0.44 4)x10^9/l.SerumIgEwas w 800(0380)kIU/m ml. The echestXray yrevealedpulmonary p infiltratesin ntherightmid m andlow werzones. Wha atisthelike elydiagnosi is?
A.Chron nicobstructivepulmonar rydisease B.Allerg gicbronchopulmonary p aspergillosis C.Pulmo onaryembol lus D.Extrin nsicallergicalveolitis a E.Mesothelioma

Answer:b)Alle ergicbronch hopulmona aryaspergill losis. ABP PA usually occurs o in association with asthm ma, but can n occur in cystic c fibros sis. It is trea ated with either e oral or inhaled steroids. There T is som me evidenc ce supporting anti fung gal agents like l itraconazole, but amphoteric a cin is not us sed. IgG and d IgE are ra aised. In additionskinte estingorex xaminationofsputumwith w fungalstainscanbeused. Sputum culture e is not ofte en helpful. ABPA can lead l to proximal bronc chiectasis affecting a theupperlobe esifleftuntreated.

CXR Rshowingpulmonaryin nfiltrateson ntherightinABPA

Dr.Khalid K Yusuf fElzohrySo ohagTeachin ngHospital 2012 43

www.MRCPass.com

Respiratory

RespiratoryQ43
A 16 year girl with know n cystic fibrosis presents with cough and fevers. Chest X ray showsrightlowerzoneconsolidation. Whatantibioticshouldbecommenced?
A.Amoxycillin B.Metronidazole C.Ceftazidime D.Gentamicin E.Piperacillin

Answer:C)Ceftazidime Patients with cystic fibrosis often have pseudomonas as a pathogenic infection. Ceftazidime and ciprofloxacin would cover the organism, and sometimes gentamicin or meropenemmayberequiredduetoresistance.
Dr.KhalidYusufElzohrySohagTeachingHospital2012 44

www w.MRCPass.c com

Res spiratory

Respiratory yQ44
A50 0yearoldasthmatic a tr reatedwithhighdoseof o inhaledcorticostero c oidsdevelop psaskin rash h. His routine blood te ests shows mild renal dysfunction and bloo od eosinoph hilia. His chestradiograp phisnorma al. ichbloodte estshouldb bedonenext? Whi
A.TotalIgElevel B.Antineutrophilcytoplasmican ntibodies C.AntiGBM G antibod dy D.Asper rgillusfumigatesprecipit tins E.Antinuclear n antib body(ANA)

Answer:b)Ant tineutrophil lcytoplasmicantibodie es. Skin n rash and renal dysf function in an asthma atic suggest ts Churg Strauss syn ndrome. Chu urgStrauss syndrome, or allergic c granulom matous angi iitis, is a ra are syndrom me that affe ectssmalltomediumsizedarteries. The e presence of 4 or more crite eria indicat tes a high likelihood of ChurgStrauss synd drome. The esecriteriaare: a (1)asthma (2)eosinop philiaofmorethan10% %inperiphe eralblood (3)paranas salsinusitis (4)pulmonaryinfiltrat tes(maybetransient) (5)histolog gicalproofo ofvasculitiswithextrav vasculareos sinophils (6)monone euritismultiplexorpolyneuropath hy. Hyp pergammaglobulinemia a, increased d immunoglobulin E (Ig gE) levels, rheumatoid r d factor, andpositiveAN NCAareusu uallypresen nt.

Pulm monaryInfiltratesinCh hurgStrausssyndrome e

Dr.Khalid K Yusuf fElzohrySo ohagTeachin ngHospital 2012 45

www.MRCPass.com

Respiratory

RespiratoryQ45
A 70 year old man with chronic obstructive lung disease presents with a cough, fevers andgreensputum. Whatistheantibioticofchoice?
A.Teichoplanin B.Cefotaxime C.Erythromycin D.Amoxicillin E.Ciprofloxacin

Answer:D)amoxicillin. InCOPD,the 3bacterial species account for mostisolates are :Haemophilusinfluenzae, Streptococcus pneumoniae and Moraxella cattarhalis. First line treatment should be withamoxicillin,butifthepatientisallergic,atetracyclineshouldbeused.
Dr.KhalidYusufElzohrySohagTeachingHospital2012 46

www w.MRCPass.c com

Res spiratory

Respiratory yQ46
A60 0yearoldpatient p wasinvolvedin nacaraccid dentandsustainedmu ultiplefractu ures.He had been in IT TU not long g before th here was a suspicion o of severe respiratory r distress drome. synd ichofthese ewouldhelp ptoconfirm mthediagno osis? Whi
A.CTsca anoftheche est B.Oesop phagealman nometry C.Pulmo onarycapilla arywedgepr ressure D.Requirementforventilatorys support E.Arteri ialbloodgas ses

Answer:c)pulm monarycap pillarywedg gepressure. Respiratory di istress synd drome can n be caused d by sever re trauma, smoke inh halation, mul ltiple blood transfusion ns, drow ning and aspi iration, and d drugs such h as salicyla ates and narc cotics. Ches st XR would d show bilateral infiltrates, and th he presenta ation mimic cs heart failu ure.Hencethe t besttes stwouldbe ePCWP,whichwouldbe b normal(< <18mmHg).

ARD DS
Dr.Khalid K Yusuf fElzohrySo ohagTeachin ngHospital 2012 47

www w.MRCPass.c com

Res spiratory

Respiratory yQ47
A 68 year old man with weight loss s has a che est x ray which w shows s a cavitating lung lesio on.Outofthe t followin ngcauses,which w ismos stlikelytocause c acavi itatinglunglesion?
A.Meso othelioma B.Smallcelllungcar rcinoma C.Squam mouscelllun ngcarcinoma a D.Adenocarcinomaofthelung E.Largecellcarcinom maofthelun ng

Answer:c)squamouscelllungcarcinoma. Squ uamous cell carcinom ma accounts for app proximately y onethird d of all ca ases of bronchogeniccarcinomas c . Squ uamous cell l carcinoma as tend to f form firm, nonencapsulated, sha arply circum mscribed mas sses located d in the ma ain, lobar or o segmenta al bronchi. Larger tum mors often outgrow o thei irvascularsupply s andmay m havecentralareasofhemorr rhage,necrosisorcavit tation.

Aca avitatingcarcinomatou uslunglesio on


Dr.Khalid K Yusuf fElzohrySo ohagTeachin ngHospital 2012 48

www.MRCPass.com

Respiratory

RespiratoryQ48
A 45 year man has a long history of productive cough. He had complained of frequent chestinfections. Examination reveals bilateral inspiratory crackles in the bases of the lungs and clubbing ofthefingers. Whichoffollowingtreatmentsislikelytoreducethefrequencyofexacerbations?
A.Prophylacticantibiotics B.Inhaledcorticosteroids C.Oralcorticosteroids D.Posturaldrainage E.Lungtransplant

Answer:D)posturaldrainage. The patient has bronchiectasis, in which the common complication is difficulty expectorating,posturaldrainageofsecretionsishelpful.
Dr.KhalidYusufElzohrySohagTeachingHospital2012 49

www.MRCPass.com

Respiratory

RespiratoryQ49
A 60 year old man, exsmoker (20 packyears), was admitted to the hospital because of a prolonged fever of up to 38.5C over a period of 10 days, associated with a progressively worsening shortness of breath, a nonproductive cough, weakness, and fatigue. The chest radiograph, showed consolidations at the base of both lungs, prominent interstitialmarkings inthemiddlelungfields, and hazyinfiltratesin the rightupperlung field. Abronchoscopywasdoneand BAL fluidanalysisshowedalveolitiswith macrophagesof 65%,lymphocytesof25%,andneutrophilsof10%. Whichdrugismostlikelytoberesponsible?
A.Sulphasalazine B.Penicillamine C.Gold D.Methotrexate E.Azathioprine

Answer:d)methotrexate. The diagnosis is interstitial pneumonitis. Methotrexate is associated with interstitial pneumonitis. This is rare but a serious complication. Diagnosis is based on the clinical setting, clinical manifestations, radiographic abnormalities, bronchoalveolar lavage (BAL),andlunghistology.
Dr.KhalidYusufElzohrySohagTeachingHospital2012 50

www.MRCPass.com

Respiratory

RespiratoryQ50
A 40 year old woman has a history of cough with copious phlegm with intermittent haemoptysis. She also gets recurrent chest infections and has a long history of joint pains.Sheisonnonsteroidalantiinflammatorydrugs. Her chest radiograph shows linear radioluciences at both bases. A highresolution computedtomographyscan(HRCT)ofthechestconfirmsbronchiectasis. Whatisthelikelycause?
A.Oldtuberculosis B.Hypogammaglobulinaemia C.Rheumatoidarthritis D.Cysticfibrosis E.Kartagener'ssyndrome

Answer:c)Rheumatoidarthritis. The case scenario suggests rheumatoid arthritis. 4 % of patients with rheumatoid arthritis develop bronchiectasis. Other causes are chronic infection (TB, measles, whooping cough etc), foreign body aspiration, hypogammaglobulinaemia, Kartagener's syndrome,Young'ssyndrome,cysticfibrosis,allergicbronchopulmonaryaspergillosis.
Dr.KhalidYusufElzohrySohagTeachingHospital2012 51

www.MRCPass.com

Respiratory

RespiratoryQ51
A35yearoldladyhasaheadache,coughandmyalgia. On examination, she has a pyrexia and scattered crackles bilaterally on auscultation of thechest.Rapidcoldagglutininswaspositive. Whatisthelikelydiagnosis?
A.Tuberculosis B.Staphylococcalpneumonia C.Streptococcalpneumonia D.Mycoplasmapneumonia E.Legionellapneumonia

Answer:d)mycoplasmapneumonia. Mycoplasmapneumoniaisthecommonestatypicalpneumonia. Approximately 15% of pneumonias in adults are due to Mycoplasma pneumoniae. Transmission occurs from person to person by infected droplets. The incubation period is 921 days. The incidence is higher during the winter months. Fever, chills, cough and headacheareearlysymptoms.Dyspnoea,chestpainandhaemoptysisarerare. Small pleural effusions may occur but are rare. Cold agglutinins are usually present in a titregreaterthan1:32.
Dr.KhalidYusufElzohrySohagTeachingHospital2012 52

www.MRCPass.com

Respiratory

RespiratoryQ52
A 32 year old man is admitted to hospital with a history of breathlessness of 46 weeks duration. He initially had flulike symptoms and was treated by his doctor with a 5day course of amoxicillin. How ever, he then started coughing up blood, leading to urgent referral. On examination he was dyspnoeic at rest, with bilateral crackles on auscultation of the lungs. Investigationrevealedanaemiaandimpairedrenalfunction(creatinine250micromol/l). Pulmonary function tests were normal apart from an abnormally high diffusion factor. Urinedipsticktestingshowedthepresenceofredbloodcells. Whatisthemostlikelydiagnosis?
A.Goodpasture'ssyndrome B.Farmer'slung C.InvasiveAspergillosis D.Extrinsicallergicalveolitis E.Chroniceosinophilicpneumonia

Answer:a)Goodpasture'ssyndrome. The clinical picture of pulmonary and renal involvement is typical of Goodpasture's syndrome.Theconditionisduetothepresenceofcirculatingantiglomerularbasement membrane antibodies (antiGBM antibodies. Other causes of pulmonary haemorrhage and renal failure include Wegener's granulomatosis, microscopic polyangiitis and systemiclupuserythematous(SLE).
Dr.KhalidYusufElzohrySohagTeachingHospital2012 53

www w.MRCPass.c com

Res spiratory

Respiratory yQ53
A 40 0 year old lady presen nts with bre eathlessnes ss, hyperven ntilation an nd tachycard dia. She hasatemperat tureof38C C.Overthelast5daysshehadbe eendeterior ratingwithacough pite a course of oral a antibiotics, but her pa artner mentions that she has and fever desp becomemuchworseover rthelastda ay. WCCis18x10^9/l CRPis220mg/l kPadespitehighflowO2 O PO2is6.5k Pulmonarywedgepressureis16m mmHg The echestXray yshowsbila ateralinters stitiallungshadowing. s Wha atisthelike elydiagnosi is?
A.Pulmonaryembolus B.Pneum mothorax C.Sever repulmonary yfibrosis D.Pulmonaryoedem ma E.Adultrespiratorydistresssynd drome

Answer:e)adu ultrespirato orydistresssyndrome. ARD DS can be precipitated p d by severe e infection, aspiration and illicit drug d use. Th he main diffe erential is pulmonary p oedema when w the Xray shows bilateral interstitial lung field shad dowing. Pu ulmonary ca apilary wedge pressure e of < 19m mmHg effect tively excludes left ventricularfailure.

Dr.Khalid K Yusuf fElzohrySo ohagTeachin ngHospital 2012 54

www.MRCPass.com

Respiratory

RespiratoryQ54
A 30 year old man with bronchial asthma was admitted to hospital for reassessment of hisrecurringdrycough,chesttightnessandwheezing. Investigationsshow: Neutrophils6(27.5)x10^9/l Lymphocytes3(1.33.5)x10^9/l Eosinophils9(0.040.44)x10^9/l CXRshowsreticulonodularshadowing Investigations to look for causes of the eosinophilia (e.g., parasitosis, immunodeficiency and malignant disease) included stool microscopy, tumour marker assays (for carcinoembryonic antigen and cancer antigen (CA) 125, CA 199, CA 153 and CA 724), mammography, gastroduodenoscopy, bronchoscopy, abdominopelvic ultrasonography, andvaginalandpelvicexamination;allyieldednormalfindings. Pulmonary function tests revealed a mild obstructive and moderate restrictive pattern [FEV1]46%,[FVC]55%,ratioofFEV1toFVC72,peakexpiratoryflow53%,[DLCO]46%. Whatisthelikelydiagnosis?
A.Tuberculosis B.Loeffler'ssyndrome C.Sarcoidosis D.Pulmonaryembolism E.Wegener'sgranulomatosis

Answer:b)Loeffler'ssyndrome. Hypereosinophilic syndrome is a rare condition where there is an idiopathic eosinophil countof>15x10^9/dl. It is associated with Loeffler's syndrome which is a transient pulmonary reaction with nodular or reticular shadowing (diffuse, fanshaped shadowing) on chest radiology and eosinophilia. Hypereosinophilic syndrome generally affects young men ages 2050. Thrombotic tendency, neurological involvement (loss of intellect, depressed mood and poor coordination)andrestrictivecardiomyopathyoccur.Thereisresponsetosteroids. The lung involvement results in nocturnal cough, productive sputum, wheezing and dyspnea,whichraisesthe suspicion of bronchial hyperreactivity. Patients may be misdiagnosed as having asthma. However,pulmonaryfunctionteststypicallyrevealnoairflowlimitation.
Dr.KhalidYusufElzohrySohagTeachingHospital2012 55

www w.MRCPass.c com

Res spiratory

Respiratory yQ55
A40 0yearoldman m hasahistory h oflongstanding gcoughwit thsputumproduction. p Healso has recurrent episodes of o chest inf fections and d sinusitis. He is married but ha as been nhavingchildren.Healsohashearingdifficul lty. unsuccessfulin Whi ichoneofth hefollowing gislikely?
A.Hypogammaglobu ulinaemia B.Alpha a1antitrypsi indeficiency y C.Defec ctinCFTRgene D.Karta aganer'ssynd drome E.Asthm mavariant

Answer:d)Kart tagener'ssy yndrome. Kart tagener's syndrome is heredi itary. It comprises c a triad of: o situs inversus (transposition) of the viscera, v ab bnormal fro ontal sinus ses produc cing sinusitis and bronchiectasis, ,andimmobilityofthe ecilia. Sym mptoms and d signs are dyspnoea, productive cough, rec current resp piratory inf fections, rheu umatoidart thritis,rena alabnormalities,malfo ormationsof frenalvess selsandano omalous subclavian arte ery. There is also oti itis media, nasal spee ech, conduc ctive hearing loss, ano osmiaorclubbing.

Dex xtrocardiain nKartagene er'ssyndrom me

Dr.Khalid K Yusuf fElzohrySo ohagTeachin ngHospital 2012 56

www w.MRCPass.c com

Res spiratory

Respiratory yQ56
A 40 0 year old woman w is referred wit th a history y of red, pai inful legs of f 3 weeks duration d that t have not responded to a course e of fluclox xacillin given n for celluli itis. She is afebrile, a doe es not have any other s symptoms, and has ne ever smoked d. Examinat tion reveals s tender purple/redno odulesonhe ershins.Ac chestradiog graphshow wsprominen nthilarregio ons. atistheapp propriatemanagemen m nt? Wha
A.LungBiopsy B.Arran ngeCTscano ofthelungs C.Arran ngebronchos scopyandbr ronchoalveolarlavagetoexcludemalignancy D.Startprednisolone E.Arrangeoutpatien ntsappointm mentforfollo owup

Answer:e)arra angeoutpat tientsappointmentfor rfollowup. The e combinat tion of bilateral hila ar lymphad denopathy and eryth hema nodo osum is diag gnostic of sarcoidosis s . This is usu ually selflim miting. She should how ever be seen in outpatients wi ith full lung g function tests including transf fer factor and a lung volumes. Seru um angiote ensinconve erting enzym me (ACE) le evel and lu ung function ns can be used to mon nitordiseas se.Worseningdiseasecanbetreatedwithpr rednisolone e.

Sarc coidosis
Dr.Khalid K Yusuf fElzohrySo ohagTeachin ngHospital 2012 57

www.MRCPass.com

Respiratory

RespiratoryQ57
A 50 year old patient has become more breathless over several days. He was assessed withaspectrumoftests.ThechestXraywasnormal.Intheinterpretationofhisarterial bloodgasesonairthefollowingresultswereobtained: pO2:8.0kPa pCO2:9.2kPa pH:7.40 BaseExcess+2 Whatdiseaseislikelytohavecausedthis?
A.Communityacquiredpneumonia B.Bronchiectasis C.Tuberculosis D.Guillainbarresyndrome E.Smallcellcarcinoma

Answer:d)guillainbarresyndrome. The raised CO2 and hypoxia demonstrate type II respiratory failure. The causes of this couldbeobstructivelungdisease,neurogenic,ormusculoskeletal(kyphoscoliosis).
Dr.KhalidYusufElzohrySohagTeachingHospital2012 58

www.MRCPass.com

Respiratory

RespiratoryQ58
A 40 year lady presents with a 6 month history of intermittent haemoptysis. She describesepisodesofcutaneousflushing,whichtypicallyaffectstheheadandneck.The episodesareoftenassociatedwithanunpleasantwarmfeeling,itchingfeeling.Theylast forhalfanhour. She is slightly breathless but has had no evidence of leg swelling or chest pain. She smokes10cigarettesaday. She has lost 3 kg of weight in the past two months and has been experiencing night sweats.AChestxrayshowssailsignintheleftlowerlobe,butnovisibleconsolidation. Whichoneofthefollowingisthemostlikelydiagnosis?
A.Wegener'sgranulomatosis B.Goodpasture'ssyndrome C.Carcinoidtumour D.Sarcoidosis E.Tuberculosis

Answer:c)carcinoidtumour. A sail sign on the CXR indicates collapse of the left lower lobe. The likely diagnosis is a carcinoidtumour,whichisassociatedwithsmoking. An early and frequent (94%) symptom of carcinoid tumors is cutaneous flushing, which typically affects the head and neck. Episodes are often associated with an unpleasant warm feeling, itching, palpitation, upperbody erythema and edema, salivation, diaphoresis, lacrimation, and diarrhea. Exercise, stress, or certain foods (eg, cheese) maytriggeranattack.Initialattacksareshort,lastingonlyafewminutes.Withtime,the duration increases to hours. Flushes are reported to be longest in association with bronchialcarcinoids. Carcinoidtumours areneuroendocrine tumoursarisingfromKultchitzskycells. They can be central or peripheral, and are classified as typical or atypical depending on their histology. They are slow growing tumours of lung with a peak incidence around age 40. Bronchialobstructioniscommon.Diagnosisisbybronchoscopyandbiopsy.
Dr.KhalidYusufElzohrySohagTeachingHospital2012 59

www w.MRCPass.c com

Res spiratory

Trea atmentisus suallywithlobectomy. .

Sailsign(straig ghtlinearou undthehea artborder)

Dr.Khalid K Yusuf fElzohrySo ohagTeachin ngHospital 2012 60

www.MRCPass.com

Respiratory

RespiratoryQ59
A 68 year old man with emphysema is referred to a chest clinic for consideration of oxygentherapy.Despitemaximaltreatmentwithbronchodilators,hisexercisetolerance isreducedtoabout25yards. Physical examination and pulmonary function test was consistent with emphysema. Therewasnoevidenceofheartfailure.ECGwasnormal.CXRshowedhyperinflatedlung fields. Oxygensaturationwas90%.ABGshowedphof7.36,pCO2of3.7kPa,pO2of7.6kPa. Whatshouldberecommended?
A.OvernightSaO2monitoring B.Courseofsteroids C.Walkingtestwithatrialofoxygen D.RepeatedABGin6weekstime E.Longtermoxygentherapy

Answer:c)Walkingtestwithatrialofoxygen. Thispatientdoesnotmeetthecriteriaforlongtermoxygentherapy,whichisindicated when the paO2 is persistently below 7.3 kPa on air. Clinical stability is defined as the absenceofexacerbationofchroniclungdiseasefortheprevious5weeks. Patients without chronic hypoxaemia and not on LTOT, should be considered for ambulatoryoxygentherapyiftheyshowevidenceofexerciseoxygendesaturation(afall of SaO2 of at least 4% below 90%), improvement in exercise capacity with ambulatory oxygentherapyandmotivationtousetheambulatoryoxygenoutsidethehouse. Assessmentshouldbeperformedonbothairandsupplementaloxygenwiththepatient blindedastothecontentofthecylinder.
Dr.KhalidYusufElzohrySohagTeachingHospital2012 61

www.MRCPass.com

Respiratory

RespiratoryQ60
A 50 year old woman presents with breathlessness that has been getting gradually worseoverafewweeksandshehasdifficultymobilising.Onphysicalexaminationsheis foundtohavealargeleftsidedpleuraleffusion. Thepresenceoftheeffusionisconfirmedbychestradiography. Themostappropriateinitialinvestigationwouldbe:
A.Diagnosticaspirationofpleuralfluid B.Sputumcytology C.CTchest D.Pleuralbiopsy E.Mantouxtest

Answer:a)Diagnosticaspirationofpleuralfluid. Thefirstinvestigationshouldbediagnosticaspirationofpleuralfluid. Lights criteria can be used to distinguish transudates from exudates: in exudates at leastoneofthefollowingthreecriteriaaremet: pleuralfluidproteinconcentrationgreaterthan50%ofthatinplasma; pleuralfluidLDHgreaterthan60%ofthatinplasma; pleuralfluidLDHmorethattwothirdstheupperlimitofnormalinplasma In general, if the protein is <30g then it is considered to be a transudate, if>30gthen itisanexudate.
Dr.KhalidYusufElzohrySohagTeachingHospital2012 62

www w.MRCPass.c com

Res spiratory

Respiratory yQ61
A 38 8 year old man m is admitted to the e hospital with w a onew eek history of dyspnoea. He is HIV H positive. His chest radiograph h shows bilateral alveo olar infiltrates. The ad dmitting doctormakesadiagnosisofPneumocystiscariniipneumon nia(PCP). Wha attreatmen ntshouldbe estarted?
A.AZT etronidazole B.IVme C.IVcotrimoxazole D.IVfluconazole E.IVam mphotericin

Answer:c)IVcotrimoxazo ole. PCP P patients are usually hypoxic h and d chest radi iograph cha aracteristica ally shows bilateral b alve eolarinfiltra ates. The epatientsho ouldreceive eintravenouscotrimo oxazoleinadoseof120mg/kgforatleast 3 weeks w as well w as gluc cocorticoids s (IV methy ylpred for 3 days). Th his, when used in pati ients with moderate m to o severe hy ypoxia, decreases the risk of resp piratory failure and deathbyover50%. 5

Pne eumocystispneumonia p

Dr.Khalid K Yusuf fElzohrySo ohagTeachin ngHospital 2012 63

www.MRCPass.com

Respiratory

RespiratoryQ62
A40yearoldmanhasa3monthhistoryofcough,dyspnoeaandsputumproduction.He smokes 20 cigarettes a day and has a history of asthma. There is no history of asbestos exposure. HisWBCcountis24x10^9/lwith70%neutrophilsand3.0x10^9/l(5%)eosinophils.IgE level is elevated. He has decreased breath sounds corresponding to parenchymal infiltratesontheCXR. Whichisthebesttesttoconfirmthediagnosis?
A.HIVtest B.Stoolforova,cysts,parasites C.Autoimmunescreen D.Aspergillusprecipitins E.SputumforAcidFastBacilli

Answer:d)Aspergillusprecipitins. TheconditiondescribedisAllergicBronchoPulmonaryAspergillosis,commoneramong asthmatics and cystic fibrosis patients. Wheeze, shortness of breath and productive cougharesymptoms. Allergic bronchopulmonary aspergillosis results from an allergic reaction to Aspergillus fumigatuswhichactuallygrowsinthewallsofthebronchi. Eosinophilia and high IgE levels are suggestive of the condition. Aspergillus precipitins (labtesttodetectantibodies)thediagnosis.Thechestradiographoftenshowsevidence ofproximalbronchiectasis.
Dr.KhalidYusufElzohrySohagTeachingHospital2012 64

www.MRCPass.com

Respiratory

RespiratoryQ63
A 55 year old woman is admitted with a history of syncopal episodes and breathlessness.Thereisnopriorhistoryofrecenttravel. On examination she looks distressed. Her pulse rate is 120 beats per minute, blood pressure 85/60, JVP is elevated by 5 cm. Respiratory rate 26 per minute, there is a soft systolicmurmurattheleftsternaledge.Breathsoundsareclear. Investigationsshow:HB12.4g/dL,BloodgasespH7.42,PaCO23.3kPa,PaO28.5kPa. Shewasputonhighflowoxygenandgivenlowmolecularweightheparin. Thenextstepinmanagementofthispatientshouldbe:
A.Highdoseaspirin B.Coronaryangiography C.UrgentCTPAandconsiderthrombolysis D.Intravenousunfractionatedheparin E.VQscan

Answer:c)urgentCTPAandconsiderthrombolysis. The diagnosis massive central pulmonary embolus as the patient is hypoxic and hypotensive.ThebestmanagementistoobtainanurgentCTPAandthenthrombolyseif therearenocontraindications(e.g.highriskofbleeding).
Dr.KhalidYusufElzohrySohagTeachingHospital2012 65

www w.MRCPass.c com

Res spiratory

Respiratory yQ64
A 50 0 year old lady has bee en complaining of wor rsening difficulty with breathing over o the past t year. She has previo ously been diagnosed with a goit tre. On exa amination, she s had mar rkedstridor r. ichofthefo ollowingishelpful h ininvestigating gtheextentofairwaysobstruction n? Whi
A.FEV1 B.FVC C.Residualvolume D.Flowvolumeloop p E.Funct tionalresidua alcapacity

Answer:d)flow wvolumelo oop. The e flow volum me loop is a graphic re ecording of f airflow du uring maxim mal respirat tion and expiration at different d lung volumes s, and may y be affecte ed in a cha aracteristic way by alte erations in the t airway resistance. r Fixed lesion ns cause pla ateaus in both the inspiratory andexpiratorylimbsoftheflowvolumeloop. Variable intrathoracic lesi ions are characterized d by expirat tory slowing g and flatte ening of theexpiratorylimb.

Flow wVolumeLoop L showin ngairwayso obstruction

Dr.Khalid K Yusuf fElzohrySo ohagTeachin ngHospital 2012 66

www.MRCPass.com

Respiratory

RespiratoryQ65
A 60 year old lady has arthritic changes on her hand joints of ulnar deviation and Butonniere'sdefomirty.Shehaschronicbreathlessness. Whichofthefollowingisalikelyassociation?
A.Aspergillosis B.Pulmonaryfibrosis C.Empyema D.Lungcarcinoma E.Pulmonaryemboli

Answer:b)pulmonaryfibrosis. Thepulmonarycomplicationsofrheumatoidarthritisare: pulmonaryfibrosis(interstitiallungdisease) bronchiolitisobliteranswithorganizingpneumonia bronchiectasis interstitialpneumonitissecondarytodrugs


Dr.KhalidYusufElzohrySohagTeachingHospital2012 67

www.MRCPass.com

Respiratory

RespiratoryQ66
A 35 year old man has significant wheezing and breathlessness. Recordings of peak flowsshows diurnal variation.He was prescribed withsalbutamol but continues tohave frequentwheezyepisodes. Whatisthenextstepinmanagement?
A.Phosphodiesteraseinhibitors B.Leukotrieneantagonists C.Oralantibiotics D.Inhaledcorticosteroids E.Oralsteroids

Answer:d)inhaledcorticosteroids. Diurnal PEFR variation points tow ards a diagnosis of asthma. First line treatment are short acting B agonists such as salbutamol and Patients who do not respond to B agonists should be treated with inhaled corticosteroids (becotide or flixotide) which helptoreduceexacerbationsinthelongterm.
Dr.KhalidYusufElzohrySohagTeachingHospital2012 68

www.MRCPass.com

Respiratory

RespiratoryQ67
A 55 year old lady presents with worsening breathlessness. She has a history of Raynaudsphenomenon,heartburnandreflux. On examination, the skin over her fingers is tight and shiny. She has multiple telangiectasia over her face and nailfold capillary loops are seen. Her investigations showshehasAnticentromereantibodies. Whatrespiratorycomplicationmayoccur?
A.ChurgStrausssyndrome B.Metastaticadenocarcinoma C.Pulmonaryhypertension D.Allergicbronchopulmonaryaspergillosis E.Mesothelioma

Answer:c)pulmonaryhypertension. The patient has the clinical features of limited cutaneous scleroderma (LcSScformerly CREST syndrome). Anticentromere antibodies occur in 7080% of these patients. A significant respiratory complication is pulmonary hypertension. Treatment is with prostaglandinanaloguese.g.iloprost(intravenousinfusions)orbosentan(oral).
Dr.KhalidYusufElzohrySohagTeachingHospital2012 69

www.MRCPass.com

Respiratory

RespiratoryQ68
A16yearoldboywasbroughttotheA&Ewithwheezeandextensiverashwhilsteating ataChinesetakeaway. On examination, he had extensive wheezes in his chest, stridor, as well as urticaria coveringhisupperandlowerlimbs.HisBPis82/50mmHg. Whatisthemostlikelydiagnosis?
A.C1EsteraseDeficiency B.Salmonellainfection C.Idiopathicurticaria D.Asthma E.Allergy

Answer:E)allergy The scenario is consistent with a food allergy, e.g. nuts, leading to an anaphylactic reaction. The patient should be treated with hydrocortisone (iv or im) as well as chlorpheniramine.
Dr.KhalidYusufElzohrySohagTeachingHospital2012 70

www.MRCPass.com

Respiratory

RespiratoryQ69
A40yearoldmanwithpneumoniaisbeingexamined. Which one of the following positive auscultatory signs is diagnostic of bronchial breathing?
A.Rhonchi B.Increasedvocalresonance C.Aegophony D.Whisperingpectoriloquy E.Fineinspiratorycrepitations

Answer:d)Whisperingpectoriloquy. Whisperingpectoriloquyisadiagnosticsignforbronchialbreathing.
Dr.KhalidYusufElzohrySohagTeachingHospital2012 71

www.MRCPass.com

Respiratory

RespiratoryQ70
A 16 year old boy with previous tuberculosis had a 7day history of progressive cough, wheeze and tachypnoea, despite 4 days of intravenous flucloxacillin and cefotaxime therapy. Total serum IgE titre was 1600 IU (normal range, 0180 IU) and the skin prick test was positiveforAspergillusfumigatus. Whatshouldhebetreatedwith?
A.Praziquantel B.Aciclovir C.Itraconazole D.Rifampicin E.HAART

Answer:c)itraconazole. Allergicbronchopulmonaryaspergillosis(ABPA)isadiagnosiswhichcanbeconfirmedby significantly elevated serum IgE titre, positive skinprick tests for aspergillus, positive IgG aspergillusprecipitins. Treatment is with antifungals. Steroids may be required in patients with respiratory distress.
Dr.KhalidYusufElzohrySohagTeachingHospital2012 72

www w.MRCPass.c com

Res spiratory

Respiratory yQ71
A35 5yearoldman m whoseconditionh hassuddenlydeterioratedisbroug ghttoA&E. .Hehad arriv ved 30 min nutes earlie er with a 2hour hist tory of cen ntral pleurit tic chest pa ain and brea athlessness s.Hecollaps sedwhileaw waitingCXR. He is cyanosed d and has a pulse 130/ /min and BP B of 85/45 5 mmHg. Ox xygen satur ration is ding81%,despite d highflowoxyge enviaarebreathe b mas sk. read Respiratory ex xamination reveals reduced brea ath sounds in the rig ght lung fie eld with viationofthetracheato owardsthe eleft. dev atimmedia atecourseofactionsho ouldbetake en? Wha
A.Pleurodesis B.Insert tlargeboren needleintoright r hemithorax C.Arran ngeforurgen ntportablechestXR D.Check karterialblo oodgasesandcommence eBIPAP E.Conta actITUtoarr rangeforthe epatienttobe b ventilated d

Answer:b)Inse ertlargeboreneedlein ntorighthe emithorax. The e diagnosis is a right s sided spont taneous pneumothora ax, which has now dev veloped into oatensionpneumotho p orax. Asthe t patientis unstable, ,there is no otime toar rrangeforportable p che estXR,inse ertionof alargeborene eedlewould dreducethe epressureintheright hemithorax x.

Larg gerightside edpneumot thorax

Dr.Khalid K Yusuf fElzohrySo ohagTeachin ngHospital 2012 73

www w.MRCPass.c com

Res spiratory

Respiratory yQ72
A 45 year old man develops breathlessness an nd a non pr roductive cough. He has h mild feve ers and has lost half f a stone in weight over o the past six mo onths. He has h had prev viouslytrea atedtubercu ulosis.Heworks w asata axidriver. Ove er the past ten years he h and his p partner wen nt fossil hunting in old d quarries. Sputum sam mplesareAF FBnegative. HischestXray yshowsnod dularshadow wingintheupperzone e. atisthelike elydiagnosi is? Wha
A.Silicosis B.React tivationofTB B C.Cadm miumlung D.Histio ocytosisX E.Asbes stosis

Answer:a)silic cosis. Prev vious TB predisposes to silicosis s, which can present as fever, breathlessne b ess and weight loss. Coal C miners s, quarry w workers and d people w whose hobb bies predisp pose to exposuretosilicaareatrisk.

Silic cosis
Dr.Khalid K Yusuf fElzohrySo ohagTeachin ngHospital 2012 74

www.MRCPass.com

Respiratory

RespiratoryQ73
A 70 year retired sandblaster has worsening symptoms of cough, wheeze and breathlessness.Hehasbeenkeepingparrotsfor15yearsasahobby. SerumprecipitinsforMfaeniarenegative. ChestXrayshowshyperinflatedlungs. Hislungfunctiontestsshow: FEV11.8L(predicted2.6) FVC3.0L(predicted3.2) FEV1/FVC60% Whichofthefollowingisthelikelydiagnosis?
A.Silicosis B.Chronicobstructiveairwaysdisease C.Farmer'slung D.Allergicbronchopulmonaryaspergillosis E.Pigeonfancier'slung

Answer:b)chronicobstructiveairwaysdisease. The lung function tests show an obstructive picture (reduced FEV1/FVC ratio). Interestitial lung diseases are more likely to cause a restrictive picture on the lung functiontest.
Dr.KhalidYusufElzohrySohagTeachingHospital2012 75

www.MRCPass.com

Respiratory

RespiratoryQ74
A60yearoldasthmaticladyisadmittedwithsuddenonsetleftsidedpleuriticchestpain accompaniedbyshortnessofbreath. Arterialbloodgasesareasfollows:pHof7.30,p02of7.5kPa,andpCO2of3.8kPa. ChestXRisnormal.Sheiscommencedonoxygen. Whatisthemostappropriateimmediateaction?
A.RequestachestXRinexpiration B.RequestDdimer C.StartlowmolecularweightheparinandrequestCTpulmonaryangiography D.StartlowmolecularweightheparinandrequestaV/Qscan E.Broadspectrumantibiotics

Answer:c)StartlowmolecularweightheparinandrequestCTpulmonaryangiography. The symptoms and findings clearly point out tow ards pulmonary embolism (PE). As the clinical probability of PE is high, measurement of Ddimer should not be performed, sincetheresultwouldnotaltertheneedfordefinitiveinvestigation.MeasurementofD dimershouldonlybeperformedwhentheprobabilityofPEislow,whenanormalvalue wouldbetakenasreassuringandfurtherinvestigationwouldnotbepursued. V/Q scan less likely to be unhelpful in view of her asthma; hence a CT pulmonary angiogramwouldbetheimagingprocedureofchoice.
Dr.KhalidYusufElzohrySohagTeachingHospital2012 76

www.MRCPass.com

Respiratory

RespiratoryQ75
A 65 year old man has had 5 kg weight loss over 6 months. He is an ex smoker of 25 cigarettesadayfor40yearsandusedtoworkinacoalmine. A Chest X ray shows a large right sided pleural effusion and several pleural plaques in bothlungperipheries. Pleuralaspirationrevealsanexudatewith42gofprotein. Whatinvestigationshouldberecommended?
A.Bronchoscopy B.Lungfunctiontests C.SpiralCTofthechest D.Thoracoscopyandbiopsy E.SputumforAFB

Answer:d)thoracoscopyandbiopsy. Thispatientprobablyhasamalignanteffusionasdemonstratedbytheexudatefromthe pleural effusion. As he is symptomatic, the best option would be to drain the fluid as wellasconfirmadiagnosissimultaneously. A video assisted thoracoscopy would help to do this. In this procedure, an illuminated tube is inserted through a small incision made betw een the ribs. It allows the operator tovisualizestructuresinsidethechestandtoperformsimpleproceduressuchasbiopsy andnoduleexcision.
Dr.KhalidYusufElzohrySohagTeachingHospital2012 77

www.MRCPass.com

Respiratory

RespiratoryQ76
A 65 year old man with a smoking history of 50 a day for many years has been a chest radiographshowingalungmass.Heispresentlywaitingabronchoscopy. Whichoneofthefollowingsupportsadiagnosisofsmallcelllungcancer?
A.Disseminatedintravascularcoagulation B.Hypertrophicpulmonaryosteoarthropathy(HPOA) C.Syndromeofinappropriateantidiuretichormonesecretion(SIADH) D.Thrombocytosis E.Hypercalcaemia

Answer:c)Syndromeofinappropriateantidiuretichormonesecretion(SIADH). SIADH is most comonly seen with small cell carcinoma rather than nonsmall cell carcinoma. HPOA, hypercalcaemia without bone metastasis is more common in squamous cell carcinoma. DIC and thrombocytosis are more common with adenocarcinoma.
Dr.KhalidYusufElzohrySohagTeachingHospital2012 78

www.MRCPass.com

Respiratory

RespiratoryQ77
A 60 year old woman presents with a 6month history of progressive shortness of breath. Her past medical history is unremarkable apart from Raynauds syndrome for whichshetakesacalciumchannelblocker. Onexaminationshehastelangiectasia. Her chest radiograph shows clear lung fields, prominent pulmonary arteries and mildly enlargedheart.Spirometryisnormal,butgastransferisreducedby40%ofpredicted. Whatisthemostlikelydiagnosis?
A.Corpulmonale B.Pulmonaryarterialhypertension C.Sarcoidosis D.Pulmonaryemboli E.Pulmonaryoedema

Answer:b)Pulmonaryarterialhypertension. Thepatientislikelytohavesclerodermafromtheclinicalhistory. Thereisalsoassociatedpulmonaryhypertensionwhichsuchpatientsareatriskof.


Dr.KhalidYusufElzohrySohagTeachingHospital2012 79

www.MRCPass.com

Respiratory

RespiratoryQ78
A 75 year old lady is admitted with an acute exacerbation of chronic obstructive pulmonary disease (COPD). One hour after admission she remains distressed with a respiratory rate of 35 per minute and is peripherally cyanosed. Repeated arterial blood gasesshowasevererespiratoryacidosiswithapHof<7.2. Whichofthefollowingtreatmentsisrecommended?
A.Giveintravenousinfusionofaminophylline B.Giveintravenoushydrocortisone C.Repeatbronchodilatortherapyandarrangerepeatarterialgases D.Arrangefornoninvasivepositivepressureventilation E.Highflowoxygen

Answer:d)Arrangefornoninvasivepositivepressureventilation. Non invasive positive pressure ventilation should be considered, especially in COPD patients when there is a need for ventilatory assistance as indicated by worsening dyspnoea, acute respiratory acidosis and worsening oxygenation. If this does not work thenintubationandventilationmaybenecessary.
Dr.KhalidYusufElzohrySohagTeachingHospital2012 80

www w.MRCPass.c com

Res spiratory

Respiratory yQ79
A 65 5 year old man m presen nts with inspiratory str ridor. A che est Xray sho owed comp pression ofthetracheabyaretrost ternalgoitre. Whi ich of the following f in nvestigation ns is the most m useful to assess the severity y of his airw waysobstru uction?
A.Flowvolumeloop p B.Force edexpiratory yvolume C.Force edvitalcapac city D.Peakexpiratoryfl lowrate E.Residualvolume

Answer:A)flow wvolumelo oop The e flow volum me loop is the t best me ethod of as ssessing ext tent of obst truction ass sociated with haretroste ernalmass.

Flow wvolumelo oopshowing gairwaysobstruction

Dr.Khalid K Yusuf fElzohrySo ohagTeachin ngHospital 2012 81

www w.MRCPass.c com

Res spiratory

Respiratory yQ80
A 25 year old man who smokes, has h progressive breath hlessness. He H has had d a long history of recu urrent chest t infections s. His lung function f tests show a FEV1/FVC ratio of 65% %, reduced FEV1, FVC and KCO o of 45% pred dicted. He also has an n uncle who o had a similarpresent tation. ichisthelik kelydiagnos sis? Whi
A.Eosinophilicgranu uloma B.Asthm ma C.1an ntitrypsindef ficiency D.Fibrosingalveolitis E.Rheum matoidlung

Answer:c)1antitrypsin a deficiency. d 1 antitrypsin n deficienc cy is autos somal rece essive and causes an n emphyse ematous (obs structive picture) on the lung function f te ests, with reduced r tra ansfer factor. The phe enotypesare eMSorZ.ZZhastheworse w outco ome. Smo okingcessat tionisessen ntial.

Dr.Khalid K Yusuf fElzohrySo ohagTeachin ngHospital 2012 82

www.MRCPass.com

Respiratory

RespiratoryQ81
A 55 year old woman with rheumatoid arthritis is referred with a history of recurrent chest infections, intermittent wheeze and production of half a teacupful of sputum daily,onoccasionswithbloodstained. Whatisthemostlikelydiagnosis?
A.Pulmonaryfibrosis B.Bronchiectasis C.Emphysema D.Tuberculosis E.Squamouslungcarcinoma

Answer:b)Bronchiectasis. Bronchiectasisisassociatedwithrheumatoidarthritis,occurringin34%ofpatientswith this condition. As with all other causes of bronchiectasis, it presents with recurrent chestinfectionsandexcessivesputum.Recurrenthaemoptysisisacommonfeature.
Dr.KhalidYusufElzohrySohagTeachingHospital2012 83

www.MRCPass.com

Respiratory

RespiratoryQ82
A 38 year old man with a history of depression is brought to the accident and emergency department after being found unconscious in a garage. A friend said that he complainedofaheadache,thenhadnauseaandvomiting. Hethenbecameunrousable. Oxygensaturationsare95%onairandbreathsoundsareclearonauscultation. Whatisthelikelydiagnosis?
A.Pulmonaryeosinophilia B.Pulmonaryembolus C.Adultrespiratorydistresssyndrome D.Pneumocystispneumonia E.Carbonmonoxidepoisoning

Answer:e)Carbonmonoxidepoisoning. Carbon monoxide poisoning is produced by the incomplete combustion of carbon containingfuelssuchasgas,coal,oil,woodandcoke. Headache is the most common symptom (90%) followed by nausea & vomiting, vertigo, alterationinconsciousnessandweakness. ThecherryredskincolouroccurswhenCOHbconcentrationexceeds20%butitisrarely seen in life. Pulse oximetry gives falsely high oxygen saturation sand it is not recommended.
Dr.KhalidYusufElzohrySohagTeachingHospital2012 84

www.MRCPass.com

Respiratory

RespiratoryQ83
A 45 year man has breathlessness and a dry cough. On examination, there are bilateral basal crepitations in his lungs. Oxygen saturations are 95%on air. Circulating precipitins towardsMicropolysporafaeniarepositive. Whatisthelikelydiagnosis?
A.ABPA B.Bagassosis C.Farmer'slung D.Birdfancier'slung E.Cheeseworker'slung

Answer:c)farmer'slung. This is a form of hypersensitivity pneumonitis. Farmer's lung is caused by the actinomycetes Micropolyspora faeni, and bagassosis is caused by Thermoactinomyces sacchari.
Dr.KhalidYusufElzohrySohagTeachingHospital2012 85

www w.MRCPass.c com

Res spiratory

Respiratory yQ84
A 45 year old man develops breathlessness an nd a non pr roductive cough. He has h mild feve ers and has lost half f a stone in weight over o the past six mo onths. He has h had prev viouslytrea atedtubercu ulosis.Heworks w asata axidriver. Ove er the past ten years he h and his p partner wen nt fossil hunting in old d quarries. Sputum sam mplesareAF FBnegative. HischestXray yshowsnod dularshadow wingintheupperzone e. atisthelike elydiagnosi is? Wha
A.Silicosis B.Cadm miumlung C.Histio ocytosisX D.React tivationofTB B E.Asbes stosis

Answer:a)silic cosis. Prev vious TB predisposes to silicosis s, which can present as fever, breathlessne b ess and weight loss. Coal C miners s, quarry w workers and d people w whose hobb bies predisp pose to exposuretosilicaareatrisk.

Silic cosisMidandUpperZone Z linearand a reticulo onodularsh hadowing,


Dr.Khalid K Yusuf fElzohrySo ohagTeachin ngHospital 2012 86

www.MRCPass.com

Respiratory

RespiratoryQ85
A 70 year old man who has previously worked in the building trade presents with a history of chest pain and dyspnoea. On examination he has evidence of a rightsided pleural effusion. Pleural aspiration is performed and a pleural biopsy taken. Histology fromthebiopsyshowsmesothelioma. Whatshouldbedone?
A.Thediagnosisshouldbequeried B.Radiotherapyshouldbegiventopreventseedingoftumourcells C.Surgery D.Chemotherapyshouldbegiventopreventseedingoftumour E.Curativeradiotherapyshouldbegiven

Answer:b)Radiotherapyshouldbegiventopreventseedingoftumourcells. In mesothelioma no treatment has been shown to influence the universally fatal outcome. After obtaining a positive biopsy, radiotherapy should be given in an attempt topreventseedingoftumourcells,aroundtheareaofthebiopsy.
Dr.KhalidYusufElzohrySohagTeachingHospital2012 87

www w.MRCPass.c com

Res spiratory

Respiratory yQ86
A 50 0 year old iv drug user r has been referred r to the medica al ward afte er being bro ought in to casualty. c He e has a chr ronic cough h productiv ve of sputu um, loss of weight, an nd night swe eats.Onexa aminationhe h isunkemptandema aciated.Histracheaisdeviated d totheleft andtherearecrepitations c sovertheapexoftheleft l lung. Rshowsfibr rosisandcavitationint theleftapex. CXR elytoconfir rmthediag gnosiswould dbe: Theinvestigationmostlike
A.CTch hest B.Gastr riclavage C.Sputu umforacidand a alcoholfastbacilli D.Mant touxtest E.Fibreo opticbronch hoscopy

Answer:c)Sputumforacidandalcoh holfastbaci illi. TBislikely.Inapatientwithaproduc ctivecough, ,AFBsshou uldbepositi iveinthesp putum.

Red dcolouredAFB A

Dr.Khalid K Yusuf fElzohrySo ohagTeachin ngHospital 2012 88

www w.MRCPass.c com

Res spiratory

Respiratory yQ87
A 55 year old smoker has s a history of breathle essness and d a dry cough. He has several nod dules presen nt in the perihilar reg gion. His serum calcium m is norma al. A bronch hoscopy and transbron nchial biop psy is don ne. This shows s non n necrotic granulomas and mul ltinucleated dgiantcells. ichofthefo ollowingislikely? l Whi
A.Histoplasmosis B.Asper rgillosis C.Tuber rculosis D.Small lcellcarcinoma E.Silicos sis

Answer:c)tube erculosis. Mul ltinucleated d giant cells s are very la arge epider rmis cells th hat have mu ultiple nucle ei. They canbepresent tinviralinfections(e.g g.herpes),TB T orlymph homa.Granulomatousdisease sugg gestseither rsarcoid,TB BorWegen ner'sgranulo omatosis.

Amultinucleat m edgiantcell
Dr.Khalid K Yusuf fElzohrySo ohagTeachin ngHospital 2012 89

www w.MRCPass.c com

Res spiratory

Respiratory yQ88
A 30 year old woman with epilepsy y has been diagnosed with pulmonary tube erculosis andisaboutto obestartedonquadrupletherapy y. Whi ichdrugma aycauseuri inediscolou uration?
A.Pyraz zinamide B.Erythromycin C.Rifam mpicin D.Isonia azid E.Etham mbutol

Answer:c)Rifa ampicin. Disc coloration of o urine is due to rifa ampicin. Ri ifampicin is s a liver en nzyme indu ucer but ison niazidisanenzyme e inh hibitor.

Urin nediscolour rationinapatient p takingRifampic cin


Dr.Khalid K Yusuf fElzohrySo ohagTeachin ngHospital 2012 90

www.MRCPass.com

Respiratory

RespiratoryQ89
A30yearoldmalepatientpresentswithworseningrhinitis,coughandwheezing,which has occurred for the past 2 years. On admission, his lung function tests show FEV1 of 55%predictedandFVCof65%predictedandachestXrayshowedbilateralinfiltrates. Hehadaneosinophilcountof5x10^9/l(0.040.4)andsputumeosinophiliaof80%was also found. There was also elevated IGE levels of >1000 kU/L. Serum ANCA was positive atadilutionof1:1024. A nasal biopsy showed chronic inflammation, with some areas suggestive of vasculitis, andeosinophilicinfiltration. Whichdiagnosisismostlikely?
A.COPD B.Mesothelioma C.ChurgStrausssyndrome D.Tuberculosis E.Extrinsicallergicalveolitis

Answer:c)ChurgStrausssyndrome. There are many causes of pulmonary symptoms with eosinophilia including Loeffler's syndrome, Churg Strauss syndrome, pulmonary eosinophilic syndrome and ABPA. Extrinsic allergic alveolitis does not cause a wheeze and also does not cause eosinophilia. ChurgStrauss syndrome is an uncommon condition characterised by asthma and blood eosinophilia together with an eosinophilic vasculitis. The initial phase of the disorder is one of asthma and allergic rhinitis,often followed by peripheral bloodeosinophilia with eosinophilic tissue disease. The vasculitic phase that follows is lifethreatening; how ever, it can often be treated effectively with immunosuppression. It is associated with granuloma formation and vasculitis affecting several organs e.g. skin, pericardium, kidneyandlung.
Dr.KhalidYusufElzohrySohagTeachingHospital2012 91

www w.MRCPass.c com

Res spiratory

Seru um eosinop philia and elevated IgE levels are typical. Lab boratory dia agnosis is based on tissu ue biopsy and a the ant tineutrophi il cytoplasm mic antibody (ANCA) test. About 25% of pati ientshavecANCA c andabout50%havepANC CA.

Dr.Khalid K Yusuf fElzohrySo ohagTeachin ngHospital 2012 92

www w.MRCPass.c com

Res spiratory

Respiratory yQ90
A 44 4 year old man m has pr resented wi ith haemop ptysis. He ha ad a chest X ray which h shows mas sseswithairhaloaroundthemintheupperzones. z Hehaspositiveserumprec cipitins. Wha atisthelike elydiagnosi is?
A.tuber rculosis B.Asper rgillosis C.Actino omycosis D.Extrin nsicallergicalveolitis a E.Coalworkers w lung g

Answer:B)aspergillosis. The e air halo si ign is partic cularly asso ociated with the funga al infection n aspergillos sis, This may ybepredisp posedtoby ypreviousTB T infectionleadingtocavitation. c The T positive eserum prec cipitinsaretowardsaspergillus.

Aspergilloma

Dr.Khalid K Yusuf fElzohrySo ohagTeachin ngHospital 2012 93

www.MRCPass.com

Respiratory

RespiratoryQ91
A 55 year lady has had a long history of productive cough and shortness of breath. She often coughs up whitish sputum and is prone to chest infections. On examination her temperature is 36 C, chest expansion is reduced and there are bilateral wet inspiratory crackles. Whichoneofthefollowingtreatmentsismosthelpful?
A.Intravenousantibiotics B.Bagonistinhalers C.Posturaldrainage D.Morphine E.Prednisolone

Answer:c)posturaldrainage. This lady has bronchiectasis. There is no suggestion of a chest infection during this episodeofadmission,henceposturaldrainageisthebesttreatmentoption.
Dr.KhalidYusufElzohrySohagTeachingHospital2012 94

www.MRCPass.com

Respiratory

RespiratoryQ92
A 55 year old farmer has had progressive breathlessness over the past 2 years. He presents with a severe episode of dyspnea and productive cough which occurred 6 hoursafterhestartedworking.Hementionsthattheseepisodesaretypicalbuttodayis moresevere.Onadmission,hisO2saturationsare94%onair. CXRshowsbilateralinterstitialshadowinginupperzones. Whichofthefollowingisthemostusefultest?
A.Precipitinstoaspergillus B.SputumforAFB C.Kveimtest D.PrecipitinstoMicropolysporafaeni E.Bronchoscopy

Answer:d)precipitinstoMicropolysporafaeni. This patient is likely to have extrinsic allergic alveolitis. The classic presentation of farmer'slungresultsfrominhalationalexposuretothermophilicActinomycesspecies. Patients with extrinsic allergic alveolitis may present acutely with a flulike illness with cough. They can also present subacutely with recurrent pneumonia or chronically with exertionaldyspnea,productivecough,andweightloss. The onset of symptoms after acute exposure is usually betw een 4 and 12 hours. Some antigens provoke symptoms after repeated exposure; these include bioaerosols of microbialoranimalantigensandafewreactivechemicals. Thermophilic actinomycetes species which can cause EAA include Saccharopolyspora rectivirgula (formerly Micropolyspora faeni), Thermoactinomyces vulgaris, Thermoactinomycesviridis,andThermoactinomycessacchari.
Dr.KhalidYusufElzohrySohagTeachingHospital2012 95

www.MRCPass.com

Respiratory

RespiratoryQ93
A 60 year old woman had a CXR showing pulmonary fibrosis. Upon review , of her drug history,whichofthefollowingdrugsmightshehavebeenoninthepast?
A.Clarithromycin B.Amoxicillin C.Busulphan D.Omeprazole E.Ciprofloxacin

Answer:C)busulphan. Busulphan,bleomycin,amiodaroneandnitrofurantoinaredrugswhichcommonlycause pulmonaryfibrosis.


Dr.KhalidYusufElzohrySohagTeachingHospital2012 96

www w.MRCPass.c com

Res spiratory

Respiratory yQ94
A 65 year old farmer has s been getting worsen ning breathl lessness for r the past 4 years. The ese symptom ms occur during d work k and are often o worse e for appro oximately 10 hours afte er.Heisano onsmoker. AnXrayofthe echestreve ealedintens sifiedinterst titiallungmarkings m and dreticularchanges c in the lower parts of the lung. He also had rais sed serum precipitins to micropo olyspora ni. faen atisthedia agnosis? Wha
A.Tuberculosis B.Wege ener'sgranul lomatosis C.Churg gStrausssyn ndrome D.Extrin nsicallergicalveolitis a E.Pulmo onaryeosino ophilia

Answer:d)extrinsicallerg gicalveolitis s. Extr rinsic allerg gic alveoliti is causes a neutrophil lia due to c cell mediation but eosinophil countisnorma al.Bronchoa alveolarlava ageshowslymphocyte l esandmast tcells. EAA A is adelaye edhypersen nsitivityreaction which h may be im mmunecom mplex (III) mediated m orcell c mediate ed(typeIV)inchronicd disease. In Extrinsic Allergic Alveolitis, t the most common antigens are therm mophilic actinomycetesandavianp proteins,an ndthemost tcommondiseases d are efarmer'slu ungand bird d fancier's lung. Precip pitins to mic cropolyspor ra faeni are seen in far rmers lung g. Upper zonefibrosisca ausesaudib blecrackles. .Symptoms saretypical llyofbreath hlessnessbu utnot wheeze.

EAA Acausingup pperzonefi ibrosis


Dr.Khalid K Yusuf fElzohrySo ohagTeachin ngHospital 2012 97

www.MRCPass.com

Respiratory

RespiratoryQ95
A 22 year old woman has asthma for which she is using her salbutamol inhaler two or threetimesaday. Whatshouldbethenextstepinhermanagementifsheworsens?
A.Advisehertousethesalbutamolinhalerregularlythreetimesaday B.Addregularinhaledsteroid,e.g.beclometasone100microgtwicedaily C.Addregularsalmeteroltwicedaily D.Addregularinhaledsteroid,e.g.beclometasone1000microgtwicedaily E. Add regular inhaled steroid, e.g. beclometasone 1000 microg twice daily, plusregular longactinginhaledbetaagonist

Answer:b)Addregularinhaledsteroid,e.g.beclometasone100microgtwicedaily. TheBritishThoracicSocietyAsthmaguidelinesareasfollows: Step1:PRNuseofinhaledshortactingbetaagonists Step2:regularinhaledsteroids Step 3: highdose inhaled steroids, or low dose inhaled steroids plus longacting beta agonist Step 4: highdose inhaled steroid and regular bronchodilators (sustained release theophylline, inhaled ipatropium, oral longacting beta agonist, highdose inhaled bronchodilators, cromoglycate / nedocromil) Step5: addition of regular steroidtablets
Dr.KhalidYusufElzohrySohagTeachingHospital2012 98

www.MRCPass.com

Respiratory

RespiratoryQ96
A 60 year old woman with know n COPD is referred with a 4month history of morning headaches. She describes a dull headache which is generalised, worst upon waking up. Shehashomenebuliser(salbutamolandatrovent)butisnotonhomeoxygen. A routine arterial blood gas on admission showed a pH of 7.34 pCO2 of 6.2 kPa, pO2 of 8.8kPaandHCO330mmHg. Whatshouldbedone?
A.Longtermoxygentherapy B.Ambulatoryoxygentherapy C.CTchest D.OvernightSaO2andCO2monitoring E.RepeatABG

Answer:d)OvernightSaO2andCO2monitoring. Morning headaches are often ascribed to patients with nocturnal hypoxia or early morning hypercapnia. They are often associated with sleep apnea or a chronic respiratorydiseasesuchasCOPD. The admission blood gases show mild hypoxia, and hypercapnia. There is respiratory acidosis with metabolic compensation. It suggests that there is a high likelihood of chronichypoxia. If oxygen monitoring confirmed nocturnal hypoxia (defined as O2 sats of < 90% for morethan30%ofsleeptime),shemayrequirenocturnaloxygensupplementation.
Dr.KhalidYusufElzohrySohagTeachingHospital2012 99

www.MRCPass.com

Respiratory

RespiratoryQ97
A couple attend the GP surgery with their 2 year old daughter. She has a history of repeated chest infections, failure to thrive, and steatorrhoea (fatty stools). They also havean8yearoldchildwhoisfitandwell. Whatisthelikelydiagnosis?
A.PulmonaryEosinophilia B.Asthma C.Cysticfibrosis D.CongenitalTuberculosis E.BochdalekHernia

Answer:c)Cysticfibrosis. The gene defect in cystic fibrosis is in a mutation on chromosome 7. The inheritance is autosomal recessive so the other child without symptoms has not inherited two CF genes. Pulmonary disease develops over a few months after birth. Common infective organismsarepneumoccocus,HaemophilusinfluenzaeandPseudomonasaeruginosa.
Dr.KhalidYusufElzohrySohagTeachingHospital2012 100

www.MRCPass.com

Respiratory

RespiratoryQ98
A 45 year old man complains of wheeziness which is worse. He has a history of asthma. His current medication consists of a low dose of inhaled corticosteroids and inhaled shortactingbeta2agonisttakenthreetofourtimesaday. Whatshouldbedonenext?
A.Addoralsteroids B.Addaninhaledlongactingbeta2agonist C.Addalongactinganticholinergic D.Addashortactinganticholinergic E.Addaleucotrienereceptorantagonist

Answer:b)Addaninhaledlongactingbeta2agonist. According to the British Thoracic Society guidelines the next step would be to add an inhaled longacting beta 2 agonist (LABA) and then assess the situation. If there is a good response to LABA, this medication should be continued. If there is benefit from LABA but control is still inadequate, LABA should be continued, and the inhaled corticosteroidsshouldbeincreasedtoahighdose.
Dr.KhalidYusufElzohrySohagTeachingHospital2012 101

www.MRCPass.com

Respiratory

RespiratoryQ99
A 65 year old heavy smoker is admitted with a history of increasing breathlessness. This hasbeenprecipitatedbythedevelopmentofacoughwithyellowpurulentsputum. TheorganismsthatarelikelytocauseinfectiveexacerbationsofCOPDare:
A.Streptococcuspneumoniae,Legionellapneumophilia,Mycoplasmapneumoniae B.Streptococcuspneumoniae,Klebsiellapneumoniae,Ecoli C.Streptococcuspneumoniae,Haemophilusinfluenzae,Moxarellacatarrhalis D.Streptococcuspneumoniae,Pseudomonasaeruginosa,Staphylococcusaureus E.Streptococcuspneumoniae,Haemophilusinfluenzae,Legionellapneumophilia

Answer:c)Streptococcuspneumoniae,Haemophilusinfluenzae,Moxarellacatarrhalis. In infective exacerbations of COPD, Streptococcus pneumoniae and Haemophilus influenzae,aswellasMoraxellaarethecommonestorganisms.


Dr.KhalidYusufElzohrySohagTeachingHospital2012 102

www.MRCPass.com

Respiratory

RespiratoryQ100
A65yearoldmanhaslongstandingbreathlessnessonexertion.Hehasbeensmoking20 cigarettesadayoveraperiodof30years. On examination, the patient is breathless with use of accessory muscles and resting activationoftheabdominalmuscles.Thechestisbarrelshaped. With regards to this case, which one of the following is know n to be a predictor of mortality?
A.Spirometry B.Fibroticchanges C.BodyMassIndex D.Arterialbloodgases E.Howmanycigarettesweresmoked

Answer:c)BodyMassIndex. A low body mass index is a known predictor of mortality in patients with chronic obstructivepulmonarydisease.
Dr.KhalidYusufElzohrySohagTeachingHospital2012 103

www.MRCPass.com

Respiratory

RespiratoryQ101
A 40 year man presents with shortness of breath. On admission he is unw ell and has a respiratoryrateof32breathsperminute.Hischestiswheezyonauscultation. Arterialbloodgasesshow: pO29.5kPa,pCO23.5kPa,pH7.48,HCO325,BE+2. Whatisthelikelyclinicalscenario?
A.Lacticacidosissecondarytometformin B.Anxietydisorder C.Asthmaattack D.Chronicbronchitis E.Pneumothorax

Answer:c)asthmaattack. The patient has a respiratory alkalosis (pH > 7.45) which is acute since the bicarbonate levels are normal (2228). There is hypoxia and the patient is hyperventilating in responseandblowingoffCO2. Thiswouldbeconsistentwithanasthmaattack. Inananxietyattack,hypoxiawouldnotbepresent.
Dr.KhalidYusufElzohrySohagTeachingHospital2012 104

www.MRCPass.com

Respiratory

RespiratoryQ102
A 30 year old man is admitted with a history of haemoptysis, cough and dyspnoea. He hasbeenpreviouslyfitandhassmoked25cigarettesadayforthelast10years.Achest XR shows diffuse alveolar infiltrates. He has a microcytic, hypochromic anaemia, urine dipstick confirms haematuria and proteinuria. His lung function tests shows a normal spirometryandaTLCOof125%predicted. Whatisthemostlikelydiagnosis?
A.Pulmonaryemboli B.Chesttrauma C.Goodpasture'ssyndrome D.Pulmonarytuberculosis E.Pneumonia

Answer:c)Goodpasture'ssyndrome. Goodpasture's syndrome is characterised by diffuse alveolar haemorrhage and glomerulitis. Men are commonly affected with most cases occurring betw een the ages of 2030 years. It is more likely to occur in smokers. The anti glomerular basement antibody (AntiGBM) is present in up to 90% of the patients. Renal histology usually shows a focal segmental necrotizing glomerulitis with crescent formation. The TLCO is increased during active bleeding and can be used to monitor disease activity. An increaseabove30%ofbaselineissuggestiveofanintraalveolarbleed.
Dr.KhalidYusufElzohrySohagTeachingHospital2012 105

www.MRCPass.com

Respiratory

RespiratoryQ103
A 60 year old man presents with sever breathlessness following an upper respiratory tractinfection. Whichofthefollowingwouldsupportadiagnosisofacuterespiratorydistresssyndrome (ARDS)?
A.Highproteincontentofpulmonaryoedemafluid B.NormalchestXray C.Highpulmonarycapillarywedgepressure D.Increasedlungcompliance E.HighCO2levels

Answer:a)Highproteincontentofpulmonaryoedemafluid. Acute respiratory distress syndrome is characterised by hypoxaemia, reduced lung compliance, pulmonary hypertension and pulmonary infiltrates on chest X ray. There is damage to the capillary endothelial cell linings resulting in oedema leakage of proteins cellsintointerstitialalveolarspaces. A high pulmonary capillary wedge pressure suggests heart failure. High CO2 reflect type IIrespiratoryfailureandarenonspecific.
Dr.KhalidYusufElzohrySohagTeachingHospital2012 106

www.MRCPass.com

Respiratory

RespiratoryQ104
A 60 year old man had a transbronchial biopsy confirming squamous cell carcinoma of thelung. Whichofthefollowingisacontraindicationtowardssurgicalresection?
A.Hypercalcaemia B.Superiorvenacavalobstruction C.Previousradiotherapy D.Metastasistolocallymphnodes E.Pleuraleffusion

Answer:b)superiorvenacavalobstruction. Extensive nodal spread, distal metastases, stage IIIB or more, and superior vena caval obstructionarecontraindicationstowardssurgeryforlungcancer.
Dr.KhalidYusufElzohrySohagTeachingHospital2012 107

www.MRCPass.com

Respiratory

RespiratoryQ105
Inwhichofthefollowinghaverandomisedcontrolledtrialsshownthatlongtermoxygen therapy(LTOT)reducesmortality?
A.Asthma B.Chronicobstructivelungdisease C.Cryptogenicfibrosingalveolitis D.Cysticfibrosis E.Pulmonarysarcoidosis

Answer:B)Chronicobstructivelungdisease. There is evidence for a small reduction in mortality in patients with COPD and resting hypoxia.AlthoughindicationsforLTOTarelargelybasedonmortalitydata,somestudies have also suggested improvements in other outcome measures, including depression, cognitivefunction,qualityoflife,exercisecapability,andfrequencyofhospitalisation.
Dr.KhalidYusufElzohrySohagTeachingHospital2012 108

www w.MRCPass.c com

Res spiratory

Respiratory yQ106
A50 0yearoldwoman w hasanincident talfindingof o raisedlef fthemidiaphragmonthe t CXR. Shehashadno ohistoryofcardiothora acicsurgery yortraumatotheches st. Wha atisthelike elydiagnosi is?
A.Vagus snervepalsy y B.Horne er'ssyndrom me C.Hiatushernia D.Phren nicnervepal lsy E.Hepat tomegaly

Answer:d)phr renicnervepalsy. Cau usesofphre enicnervep palsyare: pneumonia a pleurisy aorticaneu urysm substernalgoiter neoplasms thoracicsurgery herpeszost terinfection n vasculitis diabetes

Rais sedLefthem midiaphragm m


Dr.Khalid K Yusuf fElzohrySo ohagTeachin ngHospital 2012 109

www.MRCPass.com

Respiratory

RespiratoryQ107
A 35 year old man has symptoms of wheezing and has been diagnosed as having late onset asthma. How ever, his GP measured routine blood tests and found that he had a creatinineof250umol/l. Whatantibodyislikelytobehelpfulinconfirmingthediagnosis?
A.Antinuclearantibody B.Antiphospholipidsantibody C.AntiRoantibody D.Antinuclearcytoplasmicantibody E.Antigliadinantibody

Answer:D)antinuclearcytoplasmicantibody. The clinical diagnosis is likely to be an ANCA positive small vessel vasculitis such as polyarteritis nodosa, as there is pulmonary and renal involvement. Churg Strauss syndrome should also be considered (only a small proportion of patients with Churg StrausshaveapositiveANCA)
Dr.KhalidYusufElzohrySohagTeachingHospital2012 110

www.MRCPass.com

Respiratory

RespiratoryQ108
A 60 year old man has a long history of smoking and COPD. His resting PO2 is 7.2 kPa and his continues to be breathless despite being on home nebulisers. He is assessed for longtermoxygentherapy(LTOT). WhenisLTOTindicated?
A.PO2<7.2kPa B.PO2<7.8kPa C.PO2<8kPa D.PO2<8.5kPa E.PO2<9kPa

Answer:a)pO2<7.2kPa. When thereis polycythaemiaorpulmonary hypertension,LongTerm Oxygen Therapyis indicatedwhenpO2<8kPa.InuncomplicatedCOPD,itisindicatedwhenpO2<7.2kPa


Dr.KhalidYusufElzohrySohagTeachingHospital2012 111

www.MRCPass.com

Respiratory

RespiratoryQ109
A 60 year old woman with asthma presents with a history of acute breathlessness and pleuriticchestpain.Herarterialbloodgasesshowthefollowingreadings: pH7.35,pO26.8kPa,pCO24kPa,bicarbonate25mmol/L Whichtestismostspecifictoacutepulmonaryembolism?
A.MRIofthechest B.CTpulmonaryangiogram C.Chestxray D.DDimers E.V/Qscan

Answer:b)CTpulmonaryangiogram. Most of the tests are helpful but a CT pulmonary angiogram remains the gold standard diagnostictestforpulmonaryembolism.
Dr.KhalidYusufElzohrySohagTeachingHospital2012 112

www.MRCPass.com

Respiratory

RespiratoryQ110
A 35 year old man presents with swelling of his lips and around the throat following consumptionofprawns.His investigationsshow: serumIgE150kU/L(0120) C3levelis77mg/dL(65190) C4levelis45mg/dL(1550) Whichofthefollowingdiagnosisislikely?
A.C1esteraseinhibitordeficiency B.Allergicreaction C.Systemicmastocytosis D.Moonface E.Cellulitis

Answer:b)allergicreaction. Mildly elevated IgE concentration suggests an allergic reaction to praw ns. In angioneurotic oedema due to C1 esterase inhibitor deficiency, a low C4 with normal C3 levelisseen(C2isalsolowbutnotcommonlymeasured).
Dr.KhalidYusufElzohrySohagTeachingHospital2012 113

www.MRCPass.com

Respiratory

RespiratoryQ111
A 40 year old man has with a 2 year history of increasing shortness of breath. This is worse withexertione.g. climbing the stairs at home and iseven slightly so at rest. He is anonsmoker. On examination, his pulse is 95 beats/min and his blood pressure is 140/95 mmHg. He has finger clubbing and a crackling noise at the end of inspiration over the bases of the lungs.TherewasnoankleoedemaLungfunctiontestsshow: FVC2.5l(predicted3.2) FEV12.1l(predicted2.4) FEV1%76%(predicted75%) Diffusing Capacity: Gas Transfer Factor for carbon monoxide: DLCO 17 ml/min/mmHg(predicted)25 Whichoneofthefollowingdiagnosisislikely?
A.Aspergilloma B.GuillainBarresyndrome C.Cryptogenicfibrosingalveolitis D.Asthma E.Congestivecardiacfailure

Answer:c)cryptogenicfibrosingalveolitis. A reduced FVC, with normal FEV1, FEV1% and PEF usually indicates restriction of lung volume.KCO(transferfactor)isalsoreducedinfibroticlungdisease,asinthiscase.The additional findings of inspirational crackles and clubbing suggests the diagnosis of cryptogenicfibrosingalveolitis.
Dr.KhalidYusufElzohrySohagTeachingHospital2012 114

www.MRCPass.com

Respiratory

RespiratoryQ112
A 60 year old man has severe COPD requiring home nebulisers and home oxygen. He is admittedwithaninfectiveexacerbation. Whichofthefollowingresultswouldbeexpectedonthearterialbloodgases?
A.PH7.25paCO27paO27.5HCO330 B.PH7.10paCO27paO24HCO324 C.PH7.30paCO24paO28.5HCO330 D.PH7.40paCO24paO24HCO322 E.PH7.45paCO23paO212HCO324

Answer:a)pH7.25paCO27paO27.5HCO330. A long standing COPD patient would be expected to have a high bicarbonate. However this patient is unwell with type II respiratory failure (high CO2) and hence has uncompensatedrespiratoryacidosis.
Dr.KhalidYusufElzohrySohagTeachingHospital2012 115

www w.MRCPass.c com

Res spiratory

Respiratory yQ113
A 60 0 year old miner m has been b in the occupation n for 20 yea ars. He pres sents with a cough andbreathlessness. est XR show ws diffuse in nterstitial sh hadowing. A sputum sample is po ositive for acid a fast Che bacilli. ich of the following dusts is most likely y to have predispose ed the pat tient to Whi tube erculosis?
A.Beryllium B.Cadm mium C.Coal D.Silica E.House edust

Answer:d)silic ca. Slat te workers, stonemasons and m miners are exposed e to o silica dust. Silicosis impairs mac crophagefu unction,and dinparticul lar,predisposestoTBinfection. i

Silic cosis
Dr.Khalid K Yusuf fElzohrySo ohagTeachin ngHospital 2012 116

www.MRCPass.com

Respiratory

RespiratoryQ114
A 34yearold woman presented with a dry cough, thorax constriction, and generalised weakness. During the preceding 5 months, she had experienced these dry cough episodes tw ice a week. A lung function test showed a restriction of the vital capacity (71% of the adjusted reference value), and the diffusion capacity was also reduced (66 to68%). Bronchoalveolar lavage specimens were obtained, the lymphocytes were increased up to 41% (norm, <10%), and neutrophils were increased up to 6% (norm, <2%) with a normaltotalcellcount. Whatisthelikelydiagnosis?
A.Asthma B.Pulmonaryembolus C.Tuberculosis D.Extrinsicallergicalveolitis E.Alpha1antitrypsindeficiency

Answer:d)extrinsicallergicalveolitis. Extrinsic Allergic Alveolitis is a type III or type IV response. Thereis no eosinophilia.IgG and lymphocytes are involved in immune response. Antigens of micropolyspora faeni and thermoactinomyces are 0.55 microns . These antigens which may be detected as serumprecipitins. The acuteform takes about6 hoursfor sensitisation totheinhaled antigen. Thechronic formmaytakeweeks.
Dr.KhalidYusufElzohrySohagTeachingHospital2012 117

www.MRCPass.com

Respiratory

RespiratoryQ115
A65yearoldpatientwithCOPDisonmaximaltreatment. Whichoneofthefollowingislikelytopreventfurtherdiseaseprogression?
A.Steroids B.Betaagonistinhalers C.Stoppingsmoking D.Tiotropium E.Homeoxygen

Answer:C)stoppingsmoking. In COPD, discontinuation of smoking is the only features which has been shown to reducediseaseprogression.
Dr.KhalidYusufElzohrySohagTeachingHospital2012 118

www.MRCPass.com

Respiratory

RespiratoryQ116
A60yearoldmalesmokerhasemphysema. Hislungfunctiontestsshow: FEV1is0.5(20%predicted) FVCis2.2(61%predicted) FEV1:FVCratioof26% His arterial blood gases show a PO2 of 7.5 and 7.2 on two separate occasions. He is mildlybreathlessatrest,butseverelybreathlessonexertion. Whatisthebestmeasureofhisrespiratoryfunction?
A.TLCO B.FEV1 C.FVC D.KCO E.TLC

Answer:b)FEV1. The breathlessness worsens considerably without much change in oxygen tension, suggesting that the cause of his dyspnoea is hyperinflation of his chest which worsens onexertion. Severity of emphysema is defined by the British Thoracic Society (BTS) in relation to FEV1,notFEV1:FVCratio.Mildis6080%predicted;moderate4060%andsevere<40%.
Dr.KhalidYusufElzohrySohagTeachingHospital2012 119

www.MRCPass.com

Respiratory

RespiratoryQ117
A 40 year old lady was admitted to hospital with fevers and cough productive of sputum. Chest Xray shows diffuse patchy consolidation around the left lung. She has had a flu like illnes 4 weeks ago, and has a past medical history of asthma. She also smokes 10 cigarettesaday. Whichorganismislikelytoberesponsible?
A.Mycoplasma B.Pseudomonas C.Klebsiella D.Staphylococcus E.Tuberculosis

Answer:d)staphylococcus. Following a viral infection, patients are predisposed to staphylococcal infection. The chest XR changes suggest staphlococcus rather than streptococcus (which would cause lobarconsolidation).
Dr.KhalidYusufElzohrySohagTeachingHospital2012 120

www w.MRCPass.c com

Res spiratory

Respiratory yQ118
A50 0yearoldman m presentswitha3monthhistoryofcoug ghandbreathlessness.Hewas apyrexialonad dmission. rea 7 mol/l, creatinine 100 mol/l, m sodiu um 137 His blood show ESR 60 mm/hr, ur mm mol/l,potass sium4.1mm mol/l,corre ectedcalcium2.75(2.2 22.7)mmol/l,phospha ate0.82 (0.8 81.4)mmol/ /l. Che estXraysho owsbilatera alhilarlymp phadenopat thyandegg gshellcalcification. atisthelike elydiagnosi is? Wha
A.Extrin nsicallergicalveolitis a B.Tuber rculosis C.Sarcoidosis D.Allerg gicbronchop pulmonaryas spergillosis E.Leiom myoma

Answer:c)sarc coidosis. Sarc coidosiscan ncausemanychanges sontheCXR R.Amongth heseareun nilateralorbilateral b hilar lymphade enopathy, diffuse par renchymal changes, eggshell e ca alcification, pleural effu usionsandnodules. n

Bilateralhilarly ymphadeno opathyinsa arcoidosis


Dr.Khalid K Yusuf fElzohrySo ohagTeachin ngHospital 2012 121

www.MRCPass.com

Respiratory

RespiratoryQ119
A 40 year old woman presents with breathlessness, cough and fever. On examination, she has basal crackles in the lung fields. Circulating precipitans to Micropolyspora faeni arepositive. Whichofthefollowingisthemostlikelydiagnosis?
A.Pigeonfanciers'lung B.AllergicBronchopulmonaryAspergillosis C.Farmers'lung D.PCPinfection E.Pulmonaryfibrosis

Answer:c)Farmers'lung. Farmer's lung is the most common type of hypersensitivity pneumonitis. Hypersensitivity pneumonitis, also known as extrinsic allergic alveolitis, is associated with intense or repeated exposure to inhaled biologic dusts. The classic presentation of farmer's lung results from inhalational exposure to thermophilic Actinomyces species andoccasionallyfromexposuretovariousAspergillusspecies. Acute farmer's lung develops after large exposure to moldy hay or contaminated compost. Symptoms often spontaneously resolve within 12 hours to days if antigen exposure is eliminated or avoided. It manifests as new onset of fever, chills, nonproductivecough,chesttightness,dyspnea,headache,andmalaise.
Dr.KhalidYusufElzohrySohagTeachingHospital2012 122

www.MRCPass.com

Respiratory

RespiratoryQ120
A 50 year old meat factory worker has been unw ell with fever, cough, sweats and lethargy.Onexamination,hehadapurpuricrashandhepatosplenomegaly. Whatisthelikelydiagnosis?
A.Pulmonaryembolism B.Respiratorysyncytialviralpneumonia C.Streptococcalpneumonia D.Qfeverpneumonia E.Aspergilloma

Answer:d)Qfeverpneumonia. Q fever is due to Coxiella burnetii and is acquired via animal contact. It can occur in outbreaks in farming communities and in abbatoirs. Treatment is with prolonged courses of tetracyclines. Rarely infection can be persistent leading to chronic symptoms including fatigue, malaise and sweats. Hepatitis, hepatosplenomegaly, maculopapular rashandendocarditisareassociated.
Dr.KhalidYusufElzohrySohagTeachingHospital2012 123

www w.MRCPass.c com

Res spiratory

Respiratory yQ121

A 50 year old woman, is admitted w with malais se and feve er. Four weeks previou usly she had suffered a chest infec ction for wh hich she wa as given a course of or ral amoxycillin. She felt better init tially, but is s now getti ing worse, complainin ng of interm mittent feve ers. Her chest radiograph shows a left sided d pleural effusion. An n aspirate shows s fluid with a prot teinof42gandpHof7.0. ydiagnosisis: i Themostlikely
A.Collap pseoftheleftlung B.Empy yema C.Serou uspleuraleff fusion D.Fibroticlungdisea ase E.Tuber rculosis

Answer:b)Emp pyema. The e likely cau use is a pneumonia p which has s not reso olved, leading to em mpyema. Examination of a pleural aspirate (m microscopy for organis sms, cultur re). A prote ein level >30 0gwouldbeconsistentwithanexu udateandapHof<7.2 2issuggest tiveofinfec ction.

Emp pyema(airfluid f level)

Dr.Khalid K Yusuf fElzohrySo ohagTeachin ngHospital 2012 124

You might also like